Вы находитесь на странице: 1из 84

FOREWORD

This textbook is compiled to meet the needs of students taking a TOEIC course which forms part of the curricula at Lac Hong University.

In selecting the materials for this textbook, the compilers set before themselves a modest goal. They were guided by the desire to provide the students, within the framework of the textbook, the possibility of getting acquainted with the format of the new TOEIC test.

We are aware of the deficiencies that remain in the textbook in spite of the efforts to make it an efficient aid in the study of this TOEIC course. Needless to say, the compilers will be greatly indebted for all criticism and corrections sent to Lac Hong TOEIC Center via the following e-mail address toeiclachong@yahoo.com .

TABLE OF CONTENTS
Page TEACHING PLAN
........................................................................................................................ ...........................

3 4 5

END-OF-TERM TEST FORMAT AND COURSE RESULT UNIT 1 UNIT 2 UNIT 3 UNIT 4 UNIT 5 UNIT 6 UNIT 7 UNIT 8 UNIT 9

............................................................................................................................................. .......................................................................................................................................... .......................................................................................................................................... .......................................................................................................................................... .......................................................................................................................................... .......................................................................................................................................... .......................................................................................................................................... .......................................................................................................................................... .......................................................................................................................................... ....................................................................................................................................... ....................................................................................................................................... ....................................................................................................................................... ....................................................................................................................................... ....................................................................................................................................... ...............................................................................................................

10 15 20 25 30 36 41 47 51 56 60 64 68 72 72 73 74

UNIT 10 UNIT 11 UNIT 12 UNIT 13 UNIT 14

LANGUAGE FOCUS Auxiliary Verbs

........................................................................................................................ ............................................................................................................ .......................................................................................................

English Verb Tenses

Infinitives and Gerunds

Participles and Participle Clauses ................................................................................. 75 Negation and Parallel Structure


.....................................................................................

76

Comparisons ............................................................................................................................... 77 Agreement .................................................................................................................................... 78 Relative Clauses ....................................................................................................................... 79 Modification and Word Order
........................................................................................

80

Indefinite Pronouns ................................................................................................................ 81 Active Voice and Passive Voice Conjunctions and Prepositions REFERENCES
...................................................................................

82 83 84

.......................................................................................

.................................................................................................................................

TEACHING PLAN TOEIC PREPARATION COURSE TERM 2 (15 weeks x 4 periods = 60 periods)
INCOMPLETE SENTENCES ( VOCABULARY & GRAMMAR) PART B INCOMPLETE TEXTS READING COMPREHENSION SPEAKING PART E

WEEK

UNIT

LISTENING

UNIT 1

PART A

PART C

PART D

TOPIC 1.1

UNIT 2

PART A

PART B

PART C

PART D

TOPIC 1.2

UNIT 3

PART A

PART B

PART C

PART D

TOPIC 1.3

UNIT 4

PART A

PART B

PART C

PART D

TOPIC 1.4

UNIT 5

PART A

PART B

PART C

PART D

TOPIC 1.5

UNIT 6

PART A

PART B

PART C

PART D

TOPIC 1.6

UNIT 7

PART A

PART B

PART C

PART D

TOPIC 1.7

UNIT 8

PART A

PART B

PART C

PART D

Speaking

UNIT 9

PART A

PART B

PART C

PART D

Speaking

10

UNIT 10

PART A

PART B

PART C

PART D

Speaking

11

UNIT 11

PART A

PART B

PART C

PART D

Speaking

12

UNIT 12

PART A

PART B

PART C

PART D

Speaking

13

UNIT 13

PART A

PART B

PART C

PART D PART D

- Class attendance (10%) and 30% marks announced - Submission of score-sheets with students signatures to LH TOEIC Center

14 15

UNIT 14

PART A

PART B REVIEW

PART C

END-OF-TERM TEST FORMAT


At the end of the term, the students will take a multiple choice test which accounts for 60% of the course result. This test lasts 70 minutes and consists of 7 parts as follows:
SECTION A: LISTENING: 40 Questions (5.0 marks) Listen just one time PART 1: Picture Description: PART 2: Questions and Responses: PART 3: Short Conversations: PART 4: Short Talks: 04 Questions 12 Questions 12 Questions 12 Questions

SECTION B: GRAMMAR AND READING COMPREHENSION: 40 Questions (5.0 marks) PART 5: Grammar and Vocabulary: PART 6: Incomplete Texts: PART 7: Reading Comprehension: 20 Questions 09 Questions 11 Questions

TOTAL NUMBER OF QUESTIONS:

80 (0.125 MARK/CORRECT ANSWER)

UNIT 1
PART A: LISTENING Part 1: Picture Description Listen and choose the statement that best describes the picture. 1. (A) (B) (C) (D) 4. (A) (B) (C) (D)

2. (A) (B) (C) (D)

5. (A) (B) (C) (D)

3. (A) (B) (C) (D)

Part 2: Questions and Responses Listen and choose the correct answer. 1. 2. 3. (A) (A) (A) (B) (B) (B) (C) (C) (C) 4. 5. (A) (A) (B) (B) (C) (C)

PART B: VOCABULARY AND GRAMMAR (Auxiliary verbs) Choose the correct word or phrase to complete each sentence. 1. Ken should have studied harder; he .the test again. A. succeeded in B. passed C. failed D. rejected 2. Im . Would you lend me some money? A. poor B. rich C. broke D. broken 3. Please keep this medicine refrigerated. Do not it in temperatures above 15 degrees Celsius. A. remain B. store C. have D. take 4. Madeline must be very tired; she is on the sofa. A. jumping B. lying C. awake D. laying 5. When he was ., he would sit under this tree. A. young B. youth C. childhood D. child 6. The government wants to accelerate theof Internet commerce. A. grow B. grows C. growing D. growth 7. Take the time to fully prepare yourself for interview. A. succeed B. succeeding C. success D. succession 8. , America depends on Africa for 40% of its oil imports. A. Interest B. Interesting C. interested D. Interestingly 9. Sustainable is a key indicator of long-term health of the economy. A. produce B. to produce C. product D. productivity 10. What did he her to do this morning. A. say B. tell C. speak D. question 11. Must I lock the door before I leave? No, you ........... . My son can do it for you. A. mustn't B. needn't to C. don't D. dont have to 12. I go to the post office this morning. A. ought B. have C. must D. used to 13. The black leather jacket ..be Harleys. A. isnt B. cant C. not must D. shant 14. I dont think it will rain. However, if it .., turn off the machine. A. do rain B. do rains C. does rain D. doesnt rain 15. Id rather .. shopping tomorrow. A. going B. go C. went D. to go 16. If he had had time, he joined us. A. can have B. had C. could have D. certainly had 17. He asked her, you like some more coffee? A. Will B. Shall C. Would D. Do 18. The baby be hungry because he has just had milk. A. cannot B. must C. isnt D. wasnt 19. Before she died, her husband .to take a walk with her daily. A. used B. ought C. might D. should 20. You .tell a lie. A. had not better B. had better not C. not had better D. had better not to

PART C: INCOMPLETE TEXTS Choose the best word or phrase for each blank. Questions 1 through 4 refer to the following information. It is important to stretch both before and after exercising. Many fitness experts say that stretching after you exercise is actually more important than before. However, many people do not bother stretching after their workout. They say that they are too (1) -------, or they just forget. Stretching has many benefits. (A) tiring (B) tire (C) tired (D) tires For example, it helps you avoid painful cramps in your muscles the next days. It helps to (2) ------- your flexibility. If you stretch after every workout, it (3) -----(A) improve (A) can (B) make (B) mustnt be (C) exercise (C) will be (D) stimulate (D) might easy to touch your toes. You might be surprised to know how many people (4) ------- reach their toes. Finally, it is also a good way to relax and wind down (A) must (B) cant (C) want (D) should after strenuous exercise. So be sure to include ten minutes of stretching as part of the start and end of your exercise routine. Questions 5 through 8 refer to the following notice.

Red Cross December Blood Drive


The Red Cross (5) ------- like to thank its regular donors for all their contributions. (A) will (B) could (C) would (D) can Without your help, we could not continue with our good work. Every pint of blood that you donate allows us to help people in need. Your blood saves lives. Our next blood drive will (6) ------- on December 22nd, from 9. a.m. to 7 p.m. It will be held at (A) hold (B) be held (C) held (D) be holding the Red Cross Hall on Lion Street. We are staying open later than (7) ------- so that (A) regularly (B) usual (C) regular (D) usually working people can drop by after work. If you have donated before, please bring your registration card to save time. First time donors are also more than welcome. Please encourage your friends and family members to come along, too. Giving up less than one hour of their time could give many more years of life to another (8) -------. (A) persons (B) people (C) person (D) peoples

PART D: READING COMPREHENSION Choose the best answer. Questions 1 through 3 refer to the following advertisement. FOR RENT: Sunny one-bedroom apartment in small, six-apartment building. $950/month. Convenient location bus, stores, restaurants nearby. Available on the fifteenth of the month. For more information call 637-1220, evenings and weekends. 1. What is being rented? (A) One bedroom (B) One apartment (C) Six apartments 2. When can a new tenant move in? (A) On the weekend (C) In fifteen days (D) $950

(B) On the fifteenth day of the month (D) As soon as the rent is paid

3. What does the ad tell us about the place for rent? (A) It is small. (B) It is cheaper than nearby apartments. (C) It is close to restaurants and stores. (D) It is available this weekend. Questions 4 through 6 refer to the following advertisement. Hamburger House Summer Special Offer! That's right; we've brought back our annual Summer Special Offer here at Hamburger House. Buy two juicy hamburgers get one hamburger free! Buy two giant cheeseburgers get one cheeseburger and one drink free! Cool off with a Mountain Milkshake (Caramel or Raspberry) for only $1.50 or a Glacier Sundae (Blueberry or Double Chocolate) for only $2.50. The Summer Special Offer will last all summer, from June 1st to August 31st! 4. If you buy two giant cheeseburgers, what do you get free? (A) Two hamburgers and a drink (B) One cheeseburger and one drink (C) Two cheeseburgers and a drink (D) One cheeseburgers and two drinks 5. How long does the offer last? (A) All year (B) All month (C) Three months (D) Three weeks

6. How often does this special offer happen? (A) Every year (B) Once a month (C) Not very often (D) Twice a year

PART E: SPEAKING TOPIC 2.1: CLOTHES AND FASHION Suggested questions: 1. About how much money do you spend on clothes a year?
........................................................................................................................................................................................................................................................

2. Do you like shopping for new clothes? Why or why not?


........................................................................................................................................................................................................................................................

3. What colors do you think look good on you? Why?


........................................................................................................................................................................................................................................................

4. Do you think people feel different when they wear different clothes? Why/why not?
........................................................................................................................................................................................................................................................

5. Have you ever made your own clothes? If yes, what did you make?
........................................................................................................................................................................................................................................................

6. What kinds of clothes do you think are in fashion now?


........................................................................................................................................................................................................................................................

7. What do you think of women who wear short mini-skirts?


........................................................................................................................................................................................................................................................

8. Where are some good stores to buy clothes in this area? Whats your favorite store?
........................................................................................................................................................................................................................................................

9. What do you think about second - hand clothes? Why?


........................................................................................................................................................................................................................................................

10. How do you think about teenagers fashion nowadays?


........................................................................................................................................................................................................................................................

UNIT 2
PART A: LISTENING Part 3: Short Conversations Listen and choose the correct answer. 1. What feature does the man mention? (A) The woman's height (B) The woman's age (C) The woman's face (D) The woman's hair 2. What is the woman doing? (A) She is fixing the photocopier. (B) She is using the fax machine. (C) She is making photocopies. (D) She is taking photos. 3. Which sentence is true? (A) Mary doesn't know the woman. (B) The photocopies are ready. (C) The woman is a new employee. (D) The man wants to make photocopies. 4. Why does the woman have a headache? (A) The room is very large. (B) The room is dark. (C) The man doesn't want to work. (D) The woman works too hard. 5. What warning does the man give the woman? (A) She must use less electricity. (B) She should relax more often. (C) She should not use a computer every day. (D) She should turn on the lights. 6. Who had a similar problem? (A) The man (B) The man's sister-in-law (C) The man's sister (D) The woman's sister Part 4: Short Talks Listen and choose the best answer to each question. 1. Which of the following takes the longest time to get to the speaker's workplace? (A) The subway (B) The bus (C) The bike (D) The taxi 2. How long does it take the speaker to walk to the subway station? (A) Ten minutes (B) Fifteen minutes (C) Twenty minutes (D) Fifty minutes 3. How does the speaker get to work in the winter? (A) By bus (C) By taxi (D) By bike (B) By train 4. What is the woman's job? (A) Designing cakes (B) Making wedding dresses (C) Planning weddings (D) Arranging flowers 5. Why is today an amazing day for the woman? (A) It is the woman's birthday. (B) It is her sister's birthday. (C) It is the woman's wedding. (D) Shes planning her sister's wedding. 6. Which of the following is NOT true? (A) The woman designed a cake. (B) The woman will wear a nice dress. (C) The guests will dance. (D) The woman will get married.

10

PART B: VOCABULARY AND GRAMMAR (Tenses) Choose the correct word or phrase to complete each sentence. 1. I've got a second-hand watch. A. electronic B. electric C. electrical D. electricity 2. I retired when I was 60, and now I am living on my . A. wages B. salary C. pay D. pension 3. I've run out of . Can I pay by check? A. currency B. coins C. cash D. change 4. Alex that he had taken the test five times. A. says B. tells C. said D. told 5. These pills will surely your pain. A. ease B. easy C. easily D. easiness 6. Mike didnt at Jills party last night because he had a headache. A. show up B. come up C. make up D. hang up 7. The couple must need a .because they have just returned from a long journey. A. pause B. stop C. rest D. discontinuation 8. So far, there . no word from them. A. is B. have been C. has been D. was 9. Next month when she . to Vietnam, she will come to meet me. A. return B. will return C. returns D. will have returned 10. I . to Los Angeles fifteen years ago. A. go B. was C. had gone to D. went 11. Ever since he arrived, he .quietly in the corner. A. sat B. has been sitting C. sits D. is sitting 12. Kate usually ..to class by bicycle, but today she went by bus because of the rain. A. go B. goes C. went D. gone 13. Frank ..a TV show when Karen got home. A. watched B. watches C. watch D. was watching 14. We .her for more than twenty years. A. know B. have been knowing C. have known D. knew 15. His father ..a lawyer, but now hes politician. A. is used to be B. was used to be C. use to be D. used to be 16. We were asked not to disturb the baby because he . . A. is sleeping B. slept C. was sleeping D. sleeps 17. He has studied English he was in elementary school. A. during B. while C. as long as D. since 18. While Steve was washing his car, he some dents in the doors. A. discovered B. is discovering C. was discovering D. has discovered 19. By the time he arrived at the , the movie had started. A. doctors office B. home C. cinema D. dentist 20. Almost every part of our livescomputerized over the past 10 years. A. have been B. has been C. was D. had done

11

PART C: INCOMPLETE TEXTS Choose the best word or phrase for each blank. Questions 1 through 4 refer to the following letter.
624 South Wells Street Reno, Nevada 89400 Mr. Norm Thompson 97 Vine Circle Reno, Nevada 89400

Dear Mr. Thompson: I want to rent an apartment. My friend says that you are a good (1) -------, and that (A) landlord (B) occupant (C) tenant (D) hostess you own apartments in different parts of the city. Can I rent an apartment from you? My family needs a new place to live. We love our (2) -------. Its quiet, and (A) neighbor (B) neighborly (C) neighboring (D) neighborhood its close to my job. However, our apartment is ( 3) ------ small for us. There are (A) much (B) too (C) enough (D) some four of us: my wife, our two children, and me. We need a larger apartment. We are looking for one with three bedrooms and a large kitchen. We live near Plumas Pass, and we would like to stay in this area. If you have an apartment in Plumas Pass that is (4) -------now, please let me know. (A) distant (B) occupied (C) available (D) expensive Thank you for your help. Sincerely, Fabian Ricardo Questions 5 through 8 refer to the following letter. Dear Ms. Whitfield, Let me answer your questions (5) ------- our scholarship policy. In order to qualify for (A) regards (B) to regards (C) regarding (D) my regards to a scholarship, students have to receive at least an A in 90% of their assignments, and no lower than a B+ in the remaining 10%. This is the minimum requirement. Getting the minimum grades does not (6) ------- a scholarship if there are several students (A) promise (B) agree (C) guarantee (D) offer whose grades exceed the minimum. Secondly, students cannot receive a scholarship more than two times. In addition to this, the scholarships cannot be for two consecutive semesters. You (7) ------- a scholarship last semester, so (8) -------, we were unable to offer you (A) were received (A) coincidentally (B) received (B) unfortunately (C) had received (C) accidentally (D) have received (D) fortunately a scholarship even though you received an A+ in all of your courses. Sincerely, Jenifer Michaels Finance Officer

12

PART D: READING COMPREHENSION Choose the best answer. Questions 1 through 3 refer to the following advertisement. Don't spend your lunch hour waiting in line. Come to Joe's Hamburger Grill for a quick and tasty lunch. Our menu features 25 different types of hamburgers served with: cheese sliced onions or tomatoes spicy sauces and more! Your order is guaranteed to be on your table within five minutes or it's free! Joe's is located on North Main Street in the heart of the downtown business district. See you there! 1. What is being advertised? (A) A restaurant (C) A grocery store (B) A butcher's shop (D) A food delivery service

2. What does the advertisement tell us about Joe's hamburgers? (A) They are big. (B) They are served with salad. (C) The sauce costs extra. (D) There are 25 different kinds. 3. Who would most likely go to Joe's? (A) Vegetarians (B) Office workers (C) Tourists Questions 4 through 5 refer to the following note. Andrea, This note is just to remind you of what needs to be done today. I'm very busy from early this morning until late in the afternoon, so I don't have time to do all these. 1. Wash the car 2. Mail this letter 3. Pay the gas bill 4. Pick up my clothes from the dry cleaners 5. Return the videos we watched last night 6. Make a pizza for dinner tonight Thanks so much! Casey 4. Where will Andrea probably NOT go? (A) To the post office (C) To the dry cleaner (B) To the theater (D) To the car wash (D) Heart patients

5. Why does Casey ask Andrea to do these things? (A) Because he's tired (B) Because he's sick (C) Because he's not home (D) Because he's busy

13

PART E: SPEAKING TOPIC 2.2: ENTERTAINMENT Suggested questions: 1. Do you prefer listening to the radio to watching TV? Why?
........................................................................................................................................................................................................................................................

2. Do you think it is good for children to watch TV? Why or why not?
........................................................................................................................................................................................................................................................

3. What are the advantages of watching TV?


........................................................................................................................................................................................................................................................

4. What do you like to watch, cartoons or horror movies? Why?


........................................................................................................................................................................................................................................................

5. Have you ever seen the same movie more than once? Why?
........................................................................................................................................................................................................................................................

6. In your opinion, who is the most popular entertainer/comedian in Vietnam?


........................................................................................................................................................................................................................................................

7. What kind of music do you like? Why?


........................................................................................................................................................................................................................................................

8. What are some forms of gambling people play in Vietnam?


........................................................................................................................................................................................................................................................

9. What computer games have you played?


........................................................................................................................................................................................................................................................

10. Do you like TV commercials? What do you often do to entertain?


........................................................................................................................................................................................................................................................

14

UNIT 3
PART A: LISTENING Part 1: Picture Description Listen and choose the statement that best describes the picture. 1. (A) (B) (C) (D) 4. (A) (B) (C) (D)

2. (A) (B) (C) (D)

5. (A) (B) (C) (D)

3. (A) (B) (C) (D)

Part 2: Questions and Responses Listen and choose the correct answer. 1. 2. 3. (A) (A) (A) (B) (B) (B) (C) (C) (C) 4. 5. (A) (A) (B) (B) (C) (C)

15

PART B: VOCABULARY AND GRAMMAR (Gerunds/Infinitives) Choose the correct word or phrase to complete each sentence. 1. He was operated by a famous . . A. dentist B. surgeon C. doctor D. chemist 2. You should wear when riding a motorbike. A. a business suit B. a helmet C. a pair of shoes D. perfume 3. He is making every possible effort to .his opponent. A. best B. well C. good D. better 4. a smoke detector will give your family a chance to escape in the case of a fire. A. Putting B. Fixing C. Installing D. Hanging 5. Hello, can I make to see the doctor? A. a meeting B. an experiment C. an appointment D. a department 6. Many Brazilian farmers coffee on their farms. A. grow B. find C. rise D. feed 7. He was tired so he decided to down for a little while. A. lay B. lie C. stay D. remained 8. Why dont you .. your coat? It's rather hot in here. A. switch off B. take off C. turn on D. put on 9. The secretary was working that night. A. overtime B. overwork C. extra work D. over hours 10. We ..to inform you that the position has been filled. A. sorry B. apology C. apologize D. regret 11. She to giving up her project. A. protested B. complained C. suggested D. objected 12. Who is responsible for .the dishes tonight? A. washing B. to wash C. wash D. being washed 13. She forced him ..the work. A. does B. do C. did D. to do 14. Would you mind ..the window? A. to open B. opening C. to opening D. if opening 15. They needed to practise .the ball. A. to catch B. catching C. catches D. caught 16. Frank has been really busy the new products ready for the exhibition. A. getting B. get C. to get D. to getting 17. She asked which chapter.. . A. read B. reads C. to read D. reading 18. Im looking forward her tomorrow. A. to seeing B. to see C. seeing D. will see 19. It was necessary that she .her father the truth. A. tell B. to tell C. tells D. does tell 20. He urges that she . away immediately. A. goes B. go C. to go D. going

16

PART C: INCOMPLETE TEXTS Choose the best word or phrase for each blank. Questions 1 through 4 refer to the following article. Montalvo Industries announced last Friday that it would take on 100 new (1) ------- over the next six months. Our market is expanding, said company CEO (A) employees (B) merchants (C) customers (D) products Shirley Henrico, so we need to (2) ------- our production. Thats why we need to (A) grow (B) more (C) decrease (D) increase hire more workers. The company plans (3) ------ a new, larger factory on the (A) build (B) builds (C) to build (D) building outskirts of the city, which will be equipped with all the latest technology. We are building a very modern factory, said Ms. Henrico. We are very proud (4) -------. (A) of us (B) of it (C) with him (D) about it Questions 5 through 8 refer to the following information. Homeowners should (5) ------- everything they can do to protect their homes and (A) be aware (B) aware of (C) be aware of (D) be know about valuables from theft. If possible, get your neighbors to set up a neighborhood watch scheme. That way every one can keep an eye on each others house. Second ly, insurance is vital for the contents of your home. If you have some particularly valuable items, you should insure these separately. (6) ------- of your valuables and (A) Photograph (B) Picture (C) Take photos (D) Photo keep these photos somewhere safe. You can also increase your chance of stolen property being returned to you by marking your items with a fluorescent pen. The ink from the pen is invisible to the naked eye and will not (7) ------- items. However, if (A) upset (B) damage (C) regular (D) usually the police recover your property, they can scan it with a UV light and they will (8) ----- your markings. It is best to mark items with your phone number or zip code. (A) able to see (B) be able to see (C) be capable to see (D) be able of seeing Thousands of items are found by the police every year, but cant be return ed because there is no way of identifying the owners.

17

PART D: READING COMPREHENSION Choose the best answer. Questions 1 through 3 refer to the following advertisement. INSTITUTE OF INTERNATIONAL COMMUNICATION Have you always wanted to speak a foreign language? Or two or three? Now you can! We offer classes for adults in: English-French-Spanish-Japanese-Chinese-Korean All levels from beginning through advanced. Register for summer classes now. We have both day and evening schedules. Call 564-0284 M-F 7 A.M.-9:30 P.M. Or visit us at 6793 Independence Boulevard, Suite 1001. 1. What kind of business is the Institute of International Communication? (A) Language school (B) Telephone company (C) Computer training (D) Translation service 2. When is the Institute open? (A) Every day of the week (B) In the evenings only (C) Monday through Friday (D) In the mornings only 3. Who would probably not use the Institute's services? (A) Beginners (B) Children (C) Office workers (D) Adults Questions 4 through 6 refer to the following chart. Name John Andy Scott Bill Height 178 cm 175 cm 181 cm 179 cm Weight 65 kg 67 kg 79 kg 77 kg

John and his three friends decided to join the army. When they went to the army office, the army doctor began checking their height and weight. John was a bit embarrassed because he was thin and weighed only 65 kg. Andy was also embarrassed because he was the shortest. After the doctor's check-up, the boys began training. 4. Who checked their height? (A) John (B) Andy (C) The doctor (D) Bill 5. Who was embarrassed? (A) John and Scott (B) Bill and Scott (C) Andy and the doctor (D) Andy and John 6. Why did John and his friends go for a check-up? (A) Because John was sick (B) Because they were sick (C) Because they wanted to join the army (D) Because John is a doctor

18

PART E: SPEAKING TOPIC 2.3: HEALTH Suggested questions: 1. Do you exercise? If yes, what kind of exercise do you do? If no, why not?
........................................................................................................................................................................................................................................................

2. Do you go for regular medical check-ups? Why or why not?


........................................................................................................................................................................................................................................................

3. Do you take medicine when you are sick? Why or why not?
........................................................................................................................................................................................................................................................

4. Have you ever been in hospital? If yes, when and why?


........................................................................................................................................................................................................................................................

5. Do you think it is unhealthy to keep pets in your home? Why or why not?
........................................................................................................................................................................................................................................................

6. Have you ever donated blood? Why or why not?


........................................................................................................................................................................................................................................................

7. What are some things that cause stress?


........................................................................................................................................................................................................................................................

8. How can you reduce stress in your life?


........................................................................................................................................................................................................................................................

9. What kinds of food do you think are good for your health? Why?
........................................................................................................................................................................................................................................................

10. What do you think is the most serious health problem in Vietnam?
........................................................................................................................................................................................................................................................

19

UNIT 4
PART A: LISTENING

Part 3: Short Conversations Listen and choose the correct answer. 1. Which sentence is true? (A) The man and the woman were classmates. (B) The man and the woman have met before. (C) The man and the woman have never met before. (D) The man wants to buy some shoes. 2. Where did the man and the woman meet first? (A) In his shoe store (B) In the womans house (C) In the mans house (D) In a department store 3. What did the man use to do? (A) He worked in a department store. (B) He owned his own store. (C) He was an actor. (D) He had a lot of shoes. 4. Which sentence is true? (A) The man was in his office all day. (B) The man missed a meeting. (C) The man met Eric. (D) The man made a phone call. 5. Why was the man late? (A) He woke up late. (B) He had car trouble. (C) The traffic was bad. (D) His watch was broken. 6. What will the man probably do next? (A) Take the subway (B) Telephone Eric (C) Call the woman (D) Take a break Part 4: Short Talks Listen and choose the best answer to each question. 1. How often does this company have a staff picnic? (A) Every month (B) Every year (C) Twice a year (D) Every Christmas 2. Which of the following will not be available at the picnic? (A) A barbecue (C) Games (D) A movie (B) Drinks 3. Which prizes are mentioned? (A) A holiday and a CD payer (B) A DVD payer and a CD player (C) A DVD player and a holiday (D) A holiday and a television 4. Why does the speaker want to be a doctor? (A) To help sick people (B) To earn lots of money (C) To work in a hospital (D) To study at a university 5. How long was the speakers mother in the hospital ? (A) One week (B) Two weeks (C) Three weeks (D) Four weeks 6. What does the speaker say about being a doctor? (A) Its expensive. (B) His mother is a doctor. (C) Its hard work. (D) You dont have to study much .

20

PART B: VOCABULARY AND GRAMMAR (Participles and Participle Clauses) Choose the correct word or phrase to complete each sentence. 1. The firemen were able to the fire. A. turn off B. put out C. keep out D. switch on 2. Honey is often. . A. sour B. salty C. bitter D. sweet 3. For a of reasons, he wouldnt accept our offer. A. variation B. variant C. variety D. varying 4. Wholesale andsales registered negative growth last month. A. part B. retail C. division D. gross 5. In the 1930s, there was a world-wide economic . and mass unemployment. A. policy B. prosperity C. activity D. depression 6. Professor Miller requires that all papers .in ink. A. are formatted B. be copied C. be written D. made 7. The baby was about to fall asleep because the film was. . A. funny B. boring C. bored D. excited 8. The boss was to fire his secretary. A. determine B. determining C. determined D. determination 9. The movie was very . A. move B. moving C. moved D. movement 10. He has his shoes ..every day. A. put on B. shined C. made D. shone 11. This noise is very . A. annoys B. annoying C. annoyed D. annoy 12. It is not very easy to make her .once she gets upset. A. please B. pleasing C. pleased D. pleasure 13. Two teenagers were among the fourteen people.. . A. arrests B. arrest C. arresting D. arrested 14. Extremely from the hard work, he couldnt walk an inch. A. busy B. excited C. exhausting D. exhausted 15. Who is that man ..the red jacket? A. wearing B. wear C. wears D. is wearing 16. She said that the report was a bit.. A. confused B. confuses C. confusing D. confusing 17. .water is not safe for drinking. A. Polluted B. Polluting C. Pollution D. Pollutes 18. He could easily make himself.if he tried. A. understand B. is understanding C. understood D. be understood 19. Dont leave your bags .., as they may be removed without notice. A. attended B. attending C. unattended D. unattending 20. Some customers were very .about the return policy. A. confusing B. confused C. confuse D. confuses
21

PART C: INCOMPLETE TEXTS Choose the best word or phrase for each blank. Questions 1 through 4 refer to the following article. The big news in sport today is whether boxing champion Tyson Lewis will come out of retirement. Tyson Lewis was three-time world champion who quit boxing to become a childrens book (1) -------. (A) teacher (B) author (C) seller (D) reader His most well-known childrens book is The Happy Pigs Go Camping. When asked why he gave up a multi-million dolar (2) ------- in boxing to write childrens (A) job (B) sport (C) career (D) exercise books, Mr. Lewis said, Because I like kids. Tyson Lewis has been seen (3) ------ at Rocky Stallianos Fitness Center and may (A) working out (B) standing around (C) selling books (D) raising pigs be close to a return to the sport of boxing. If he returns, he will fight the current heavyweight champion Buster McFluster in Las Vegas. Can Tyson Lewis make a comeback? Many people, including all of the (4) ------- of the Happy Pig book series, hope so. (A) enemies (B) buyers (C) fans (D) opponents Questions 5 through 8 refer to the following information. The Stardust Cinema announces the third annual Festival of (5) ------- Film (A) Classic (B) National (C) Cartoon (D) International September 20 27. We will show the best films of this year from all around the world. See your favorite foreign actors perform in our comfortable, modern theater. (6) ------- will be four (A) There (B) They (C) We (D) If shows every day. Tickets are $10 a show, or $35 for four shows. Children 12 17 years old must be accompanied (7) ------- an adult. (A) to (B) by (C) for (D) with Children under 12 will not be (8) ------. (A) treated (B) educated (C) admitted (D) employed

22

PART D: READING COMPREHENSION Choose the best answer. Questions 1 through 2 refer to the following advertisement.

Happy Mart
The country's favorite discount store Don't pay more at other stores. We offer you the best prices in: Clothing for the whole family House-hold items Camping and sports equipment Gardening supplies and more!
100% customer satisfaction guaranteed or your money back.

We're open 7 days a week. Visit us soon. 1. What kind of business is Happy Mart? (A) Sports store (C) Mail order store

(B) Department store (D) Gardening store

2. What is something that you probably cannot buy at Happy Mart? (A) Dresses (B) Soccer balls (C) Brooms (D) Groceries Questions 3 through 5 refer to the following advertisement. Do you want to learn to play the guitar? Now, you can do it in just one week! For only $35, you can get our new book with two CDs. Learn to play the guitar quickly in your own home. If you are interested in lessons, we have classes at our school three times a week, from 7 p.m. to 9 p.m. Our two experienced teachers have been teaching the guitar for over ten years. For more information, call Michael at 335-4287. 3. What do you get for $35? (A) A book (C) A book and two CDs

(B) One teacher (D) A guitar

4. How long have their teachers been teaching for? (A) Five years (B) Two years (C) Ten years (D) Three years 5. How often does the school have guitar classes? (A) Once a week (B) Twice a week (C) Two hours (D) Three times a week

23

PART E: SPEAKING TOPIC 2.4: SHOPPING Suggested questions: 1. How often do you go shopping?
........................................................................................................................................................................................................................................................

2. Where is your favorite shopping area? Why?


........................................................................................................................................................................................................................................................

3. Whom do you often go shopping with?


........................................................................................................................................................................................................................................................

4. What was the last thing you bought for yourself and/or for someone else?
........................................................................................................................................................................................................................................................

5. Where should you go if you want to buy meat/presents/clothes?


........................................................................................................................................................................................................................................................

6. When you want to buy something, do you shop around and go to many stores to compare prices?
........................................................................................................................................................................................................................................................

7. If you cant find an item at the store, do you find a shop assistant and ask for help or do you leave and go somewhere else? Why?
........................................................................................................................................................................................................................................................

8. Are you a price - conscious shopper?


........................................................................................................................................................................................................................................................

9. When you buy something, what is the most important to you: price, quality or fashion? Why
........................................................................................................................................................................................................................................................

10. Which place do you prefer to go to, a market or a supermarket? Why?


........................................................................................................................................................................................................................................................

24

UNIT 5
PART A: LISTENING Part 1: Picture Description Listen and choose the statement that best describes the picture. 1. (A) (B) (C) (D) 4. (A) (B) (C) (D)

2. (A) (B) (C) (D)

5. (A) (B) (C) (D)

3. (A) (B) (C) (D)

Part 2: Questions and Responses Listen and choose the correct answer. 1. 2. 3. (A) (A) (A) (B) (B) (B) (C) (C) (C) 4. 5. (A) (A) (B) (B) (C) (C)

25

PART B: VOCABULARY AND GRAMMAR (Negation and Parallel Structure) Choose the correct word or phrase to complete each sentence. 1. A period of two weeks is called a . . A. half a week B. midday C. midnight D. fortnight 2. The for import this year has already been filled. A. quota B. quote C. quarter D. court 3. He examines the patients with great . . A. attention B. pleasure C. care D. intention 4. The clients agree to pay all hotel..on due dates. A. charges B. fines C. tuitions D. fares 5. Our basic.. is that the customer is always right. A. expertise B. policy C. profession D. specialty 6. The television..me when I study. A. pays B. distracts C. focuses D. pulls 7. He .. with me. A. never B. doesnt never go C. never goes D. does never go 8. John continued to make , though he was told not to. A. noisy B. sounded C. loud D. noise 9. what to do, he just waited until his father arrived. A. To know not B. Not to know C. No knowing D. Not knowing 10. He has traveled to Africa before, .? A. does he B. doesnt he C. has he D. hasnt he 11. Mike .repaired the car. A. did not B. does not have C. did not have D. has not 12. Im afraid we..the day very much. A. not enjoyed B. enjoyed not C. didnt enjoy D. didnt enjoyed 13. Didnt you .the exam? A. succeed B. pass C. failed D. graduate 14. I would like you go out with him at night. A. no B. not C. no to D. not to 15. He may .been there before. A. not have B. have no C. havent D. has not 16. Mr. Lee .better than he writes. A. speaks B. tells C. says D. asks 17. Roger ..nor flowers when he was in prison. A. neither saw birds B. saw birds neither C. saw neither birds D. neither sees birds 18. Venus ...the closest planet to the sun; Mercury is. A. is no B. is not C. no is D. not is 19. On my trip to Italy, I lost a suitcase, broke my glasses, and my flight home. A. miss B. missed C. did miss D. missing 20. The children enjoy playing in the sand and in the ocean. A. to swim B. swimming C. swim D. likes swimming

26

PART C: INCOMPLETE TEXTS Choose the best word or phrase for each blank. Questions 1 through 4 refer to the following article.
To: Naser Addul From: Nicolas Reed Subject: Request for office equipment

Dear Naser, My first week working for the Citron Company has been very good. I (1) -----(A) enjoyed (B) will enjoy (C) am enjoying (D) was enjoying my new job. Now I need some things for my office. Can you help me? (2) -------, there is a problem with the computer. It works very (3) ------, and it is (A) First (A) slow (B) Also (B) slowly (C) Then (C) slower (D) Finally (D) slowness difficult to get on the Internet. I think the computer doesnt have enough memory. There is also another problem with the heating system in my office. The office is very cold early in the day, but in the afternoon it is too hot. One last thing is that my desk is very small. I need a (4) ------- desk. Can you give me one? (A) biggest (B) bigger (C) smaller (D) smallest Everything else is okay. Thank you for your help, Naser. Sincerely, Nicolas Reed, Office Specialist Questions 5 through 8 refer to the following information.
To: All employees From: The Accountants Department Re: Taxes

It will soon be time to file your tax returns. We will not be issuing your financial statements automatically to (5) ------- employees as we have done. If you need a (A) all (B) every (C) all of (D) most of statement of your income for the past year, please apply (6) ------- writing to the (A) at (B) with (C) in (D) to accounting office as soon as possible. We will mail all your statements to your home address. If you wish to receive your statement at a different address, please be sure to inform us. Because we anticipate a large response to this memo, we have (7) ------- a deadline of March 15th. You must send your (8) ------ by this date, or (A) set (A) application (B) put (B) applying (C) held (C) apply (D) designed (D) applicant we cannot guarantee your statement.

27

PART D: READING COMPREHENSION Choose the best answer. Questions 1 through 2 refer to the following advertisement.

Special Sale at Bathland!


Everything that you need for your bathroom For two weeks only! 50% off our extra-large luxury bathtubs 30% off luxury soap and shampoo sets Buy two large towels and get one free Mirrors, sinks, shelves, tiles, and much more ... All up to 25% off! 1. What would you NOT buy at Bathland? (A) A television (B) Towels 2. How long will the sale last? (A) For ten days (C) Forever

(C) Soap

(D) A mirror

(B) For one month (D) For fourteen days

Questions 3 through 5 refer to the following notice. This evening's Flight 334 to Chicago is delayed due to dangerous weather conditions. Flight 334 is expected to depart tomorrow morning at 5:30 a.m. with a 7:45 a.m. arrival time in Chicago. Our gate agents will be glad to assist you with making hotel arrangements for this evening. The airline is not responsible for the hotel expenses. Any passengers wishing to make arrangements for a hotel room should speak with the gate agent on Concourse 2. 3. What is the purpose of this notice? (A) To give the Chicago weather report (B) To notify travelers that the hotel is full (C) To explain a schedule change (D) To explain how to buy a ticket to Chicago 4. When will Flight 334 arrive in Chicago? (A) This evening (B) Tomorrow evening (C) At 5:30 a.m. (D) At 7:45 a.m. 5. Who will pay for a hotel room? (A) The passengers (C) The ticket agent (B) The airline company (D) The pilot

28

PART E: SPEAKING TOPIC 2.5: MONEY Suggested questions: 1. About how much money do you usually spend each day?
........................................................................................................................................................................................................................................................

2. Do you give money to charity? Why or why not?


........................................................................................................................................................................................................................................................

3. Do your parents give you an allowance? How much? What do you use it for?
........................................................................................................................................................................................................................................................

4. Have you ever been in debt? If yes, when and why?


........................................................................................................................................................................................................................................................

5. Do you think that money can buy love? Why or why not?
........................................................................................................................................................................................................................................................

6. How important is money to you?


........................................................................................................................................................................................................................................................

7. How much do you usually spend each month on food, on transportation, and on entertainment?
........................................................................................................................................................................................................................................................

8. If someone gave you a lot of money, what would you do with it?
........................................................................................................................................................................................................................................................

9. What is the average salary in Vietnam?


........................................................................................................................................................................................................................................................

10. How do you understand the proverb: Time is money?


........................................................................................................................................................................................................................................................

29

UNIT 6
PART A: LISTENING

Part 3: Short Conversations Listen and choose the correct answer. 1. What are the man and the woman discussing? (A) A new car (B) Their manager (C) New curtains (D) A meeting 2. What color did the manager select? (A) Green and yellow (B) yellow (C) Blue (D) Green 3. Why does the man think it s a good idea? (A) The curtains will make the room look lighter. (B) The room will be less stressful. (C) Blue curtains are less expensive. (D) The room will look bigger. 4. What is the woman doing? (A) Searching for something on her desk (B) Looking for something (C) Drinking coffee (D) Looking for someone 5. Where does the man think the glasses are? (A) At home (B) On the womans desk (C) In the womans car (D) In the kitchen 6. What does the woman remember? (A) The mans birthday (B) Where she left her car (C) That her glasses are in her car (D) That her keys are on her desk Part 4: Short Talks Listen and choose the best answer to each question. 1. Why was the concert postponed? (A) Flooding (B) Not enough seats (C) Damaged lights (D) A sick singer 2. How much would five tickets cost? (A) $100 (C) $20 (D) $200 (B) $50 3. How many tickets in total are available? (A) 500 (C) 400 (D) 1000 (B) 200 4. Which three languages is the speaker studying? (A) English, Spanish, and German (B) Spanish, German, and French (C) French, English, and German (D) English, Spanish, and French 5. Why does the speaker have a lot of chances to speak Spanish? (A) The speaker is French. (B) The speaker went to Spain on vacation. (C) The speaker is Spanish. (D) The speaker has a Spanish-speaking friend. 6. Which language is the most difficult for the speaker? (A) English (B) French (C) Spanish (D) German

30

PART B: VOCABULARY AND GRAMMAR (Comparisons) Choose the correct word or phrase to complete each sentence. 1. One normally shaves with . A. a knife B. a fork C. a comb D. a razor 2. Last year Sam retired. He spent his on an expensive holiday. A. fund B. cash C. savings D. currency 3. You have to carry out this task. It is a . A. must B. want C. need D. compulsory 4. Other documents are available upon .. A. need B. want C. request D. necessity 5. The new law takes .. next week. A. outcome B. result C. effect D. consequence 6. .. must be paid to what is going on here. A. Looking B. Attention C. Presence D. Attendance 7. Prior to leaving, check your bank.. . A. change B. balance C. remains D. rest 8. This is the .hotel available in this district. A. good B. better C. best D. more good 9. Which is the ..month of the year in your country? A. hottest B. most hot C. much more hotter D. much hot 10. This jacket is . that one. A. far superior to B. more superior than C. much more superior to D. a little superior than 11. Of the two girls over there, Jane is . . A. older B. elder C. the older D. the oldest 12. Yesterdays game was..in this series. A. more exciting B. the most excited C. the most exciting D. more excited 13. The weather couldnt be ..; its perfect. A. colder B. hotter C. better D. summer 14. The longer you practise, accurately you will be able to type. A. more B. the C. the more D. the most 15. Between them, Dr. Gates has .insight. A. the greater B. greater C. greatest D. more greater 16. She tried to express her feelings she could. A. more honest than B. as honest as C. more honestly D. as honestly as 17. Of the three runners, she is by far . A. slower B. the most slowly C. very fast D. the fastest 18. Sally has been to Europe Peter has. A. the most times than B. as many times C. more times D. as many times as 19. We enjoy skiing . they do. A. as much as B. as many as C. as more as D. as more than 20. He is not a novice reporter.. . A. any longer B. some longer C. any longest D. some longest

31

PART C: INCOMPLETE TEXTS Choose the best word or phrase for each blank. Questions 1 through 4 refer to the following advertisement. Are you looking for an economical car to rent? Look no more.

Come to Mr. Misers Car Rental Agency.


Why pay more than you have to? At Mr. Misers, we have (1) ------- prices in town. Why travel farther than you (A) low (B) lower (C) the most lowest (D) the lowest have to? Mr. Misers has three convenient (2) -------. (A) cars (B) prices (C) locations (D) schedules We have offices at the airport, at the train station, and downtown on Main Street. At Mr. Misers, we (3) ------to serve you with a friendly smile. Next time, (4) -----(A) always ready are (A) rent (B) always are ready (B) rents (C) are always ready (C) renting (D) are ready always (D) will rent at Mr. Misers. Questions 5 through 8 refer to the following letter. Dear Grandma, I just wanted to (5) ------- you this email updating you on my trip, so far. (A) send (B) give (C) tell (D) prepare for As you know, I (6) ------- from Alaska on my motorcycle with the intention of riding (A) escaped (B) went out (C) set out (D) will go down from Alaska through Canada, through the West Coast of the United States and down to the bottom tip of Mexico. Right now, I am in Vancouver, Canada. Vancouver is a really beautiful city. It is very clean and the air is very (7) -------. (A) smoggy (B) dirty (C) fresh (D) airy I will probably stay here for two more days, and then continue down to Seattle, Washington. I have a friend in Seattle that I will (8) ------ with for a week, and after (A) stay (B) meet (C) be (D) see that, I will continue on my trip. I will write again soon. Love, Freddy

32

PART D: READING COMPREHENSION Choose the best answer. Questions 1 through 4 refer to the following article.
Does chicken soup cure a cold? Scientific studies have not been able to show that this is true. Nevertheless, many people use it to treat their colds. Why is this? "Because it works," says Dr. Patty Wilson of New York. "My patients always say they feel better after treating their colds with a bowl of hot chicken soup. Scientific research may not show it, but my patients understand their own experience, and they know that chicken soup makes them feel better." Whether or not chicken soup really cures colds, it does have health benefits, especially when prepared with lots of vegetables. It is nourishing and it isn't fattening. It's also easy and inexpensive to make. Sick or healthy, chicken soup makes a delicious and healthful meal for anyone.

1. What is the purpose of this article? (A) To explain how to prepare chicken soup (B) To discuss chicken soup as a treatment for colds (C) To present scientific research about chicken soup (D)To compare different treatments for colds 2. According to Dr. Wilson, why do people treat colds with chicken soup? (A) Because it makes them feel better (B) Because the doctor tells them to (C) Because it has been proven by research (D) Because it is inexpensive 3. What is a benefit of eating chicken soup? (A) It cures colds. (B) It is the cheapest food available. (C) You can buy it anywhere. (D) It is nourishing. 4. What is the writer's opinion of chicken soup? (A) It is too expensive. (B) It is a good treatment for a cold. (C) It has too much salt. (D) Only sick people should eat it.

33

Questions 5 through 8 refer to the following table and e-mail.

Asia Pacific Weather: Five-Day Forecast January 7th 11th


Beijing Tokyo Hong Kong Seoul Singapore Jakarta Sydney -4C, cloudy, with some snow 0C, sunny, with some clouds in the evening 14C, cloudy, with sunshine in the morning 3C, sunny, some light rain in the afternoon 27C, windy and rainy, with some lightning 26C, heavy rain, some lightning at night 19C, windy and cloudy all day

To: Alice Souza <aasouza@netmail.com> From: MJames@mailhouse.com Subject: Forecast Date: January 7th Dear Ms. Souza, Here is the weather forecast that you asked for. Your trip starts in Beijing. It will be very cold there, so bring lots of warm clothes. On January 10th, you will go to Singapore. It will be hot, so you need light clothes. I think you should also pack an umbrella. Have a good trip, and don't hesitate to call me if you have any questions. Sincerely, Marilyn James 5. Which city has the highest temperature? (A) Beijing (B) Seoul (C) Singapore 6. Which city has the lowest temperature? (A) Beijing (B) Seoul 7. Which city will have the most rain? (A) Hong Kong (B) Tokyo (C) Singapore (C) Jakarta

(D) Sydney (D) Sydney (D) Singapore

8. Why did Alice Souza want to know the weather forecast? (A) Because it is her hobby (B) Because she is taking a trip (C) Because she wants to buy new clothes (D) Because she doesn't like rain

34

PART E: SPEAKING TOPIC 2.6: SPORTS Suggested questions: 1. What kind of sports are you good at?
........................................................................................................................................................................................................................................................

2. Do you think everybody should practice sports? Why or why not?


........................................................................................................................................................................................................................................................

3. Were you on any sports teams in high school?


........................................................................................................................................................................................................................................................

4. Does your university have a volleyball/football team?


........................................................................................................................................................................................................................................................

5. Do you like to watch sports on TV? What kind of sports do you like to watch?
........................................................................................................................................................................................................................................................

6. What do you think of women playing football?


........................................................................................................................................................................................................................................................

7. Who is your favorite professional football player?


........................................................................................................................................................................................................................................................

8. What do you think is the most popular sport in the world? Why?
........................................................................................................................................................................................................................................................

9. Does it bother you that people gamble on sporting events? Why/why not?
........................................................................................................................................................................................................................................................

10. Is it good that professional sport is so commercial nowadays? Why/why not?


........................................................................................................................................................................................................................................................

35

UNIT 7
PART A: LISTENING Part 1: Picture Description Listen and choose the statement that best describes the picture. 1. (A) (B) (C) (D) 4. (A) (B) (C) (D)

2. (A) (B) (C) (D)

5. (A) (B) (C) (D)

3. (A) (B) (C) (D)

Part 2: Questions and Responses Listen and choose the correct answer. 1. 2. 3. (A) (A) (A) (B) (B) (B) (C) (C) (C) 4. 5. (A) (A) (B) (B) (C) (C)

36

PART B: VOCABULARY AND GRAMMAR (Agreement) Choose the correct word or phrase to complete each sentence. 1. They fell in love at first . A. sight B. look C. attempt D. glimpse 2. The television can be easily carried. It's a television. A. second-hand B. colored C. new D. portable 3. His wedding has been..in the newspaper. A. said B. told C. pronounced D. announced 4. I was.. for speeding. A. fined B. found C. founded D. find 5. I asked them to.. the cost of repairing my car. A. cost B. evaluate C. estimate D. predict 6. I wonder if you coulda room for me at the hotel. A. make B. do C. preserve D. reserve 7. Susan often.her car to school last semester. A. got on B. went with C. rode D. drove 8. There are some students who are ..to the new uniform. A. opposed B. opposite C. opposition D. for 9. .very difficult to me. A. The mathematics are B. The mathematics is C. Mathematics are D. Mathematics is 10. Writing compositionsnot easy. A. are B. is C. were D. am 11. Either Bill or we supposed to contact Sylvia about the meeting. A. is B. could C. was D. were 12. Kim Jones, together with her roommates, . to write a letter to that company. A. will B. are C. is going D. are going to 13. Most of the fish I caught yesterday.. too small to bring home. A. is B. will C. were D. was 14. Two-quarters of the land . sold to investors. A. will B. have C. have been D. has been 15. I would like to buy some of . meat. A. this B. those C. these D. a few 16. A third of the students in that class passed the test. A. have B. has C. was D. were 17. She often comes home ..two months. A. every B. another C. each D. all 18. Neither Mr. Johnson nor his colleagues .going to join the project. A. will B. is C. are D. were 19. Fifty cents ..how much I owe you. A. does B. are C. do D. is 20. There are some books on the table, whichabout fish. A. writes B. are C. is D. was

37

PART C: INCOMPLETE TEXTS Choose the best word or phrase for each blank. Questions 1 through 4 refer to the following announcement.

ATTENTION TO ALL EMPLOYEES


Our office is collecting money for the (1) ------- of last weeks terrible flood. (A) observers (B) victims (C) planners (D) reporters Many people in our city lost their homes and all their (2) ------- in the flood. (A) possesses (B) possessed (C) possessing (D) possessions We would like to send money to help them. If you wish to make a (3) -------, (A) cake (B) friend (C) salary (D) donation please bring a check or cash to Mr. Kim in Human Resources before 5:00 on Friday. We (4) ------- a check in the name of everyone in the office to the Flood Fund. (A) send (B) sent (C) will send (D) might send Questions 5 through 8 refer to the following article. Most people assume that when an artist makes a work of art, the artist intends on the artwork being around for a long time. Sculptures and paintings that have been around for hundreds of years can be still enjoyed by art (5) ------- today. (A) people (B) makers (C) lovers (D) haters But in Tibet, a special type of painting is made that is not meant to be enjoyed for very long. These are Tibetan Buddhist sand paintings. Sand paintings are usually paintings of Mandalas, or circular designs that are done using various colored (6) ------- of sand. (A) grains (B) lumps (C) tubes (D) stones Some sand paintings are quite large and may take months to complete. The paintings are usually done (7) ------- a festival. Then, on the day of the festival, the paintings (A) before (B) after (C) during (D) with are revealed. People enjoy them for a short while and then let the wind blow the sand away. Buddhists believe that sand paintings help us understand how (8) ------ our (A) dirty (B) sandy (C) exciting (D) temporary lives are.

38

PART D: READING COMPREHENSION Choose the best answer. Questions 1 through 3 refer to the following article. If you are looking for an inexpensive place to enjoy a tasty meal, try the new Caf Paris. The first thing you will notice is the interesting dcor. Paintings of flowers on the walls and fresh flowers on every table make you feel like you are in a garden in Paris. The restaurant doesn't allow smoking so the air is garden-fresh. The specialty of the house is salads and they have a wide selection. Never have I tasted such delicious salads and I plan to return to the caf several more times so that I can try them all. The soups are equally good, although the desserts could be better. But the coffee is delicious and makes a good ending to a pleasant and tasty meal at the Caf Paris. 1. What is the purpose of this article? (A) To review a restaurant (B) To describe Parisian food (C) To discuss Parisian art (D) To explain the writer's food preferences 2. What is the writer's opinion of the Caf Paris? (A) The walls need fresh paint. (B) The salads are very good. (C) The soups are better than the salads. (D) The desserts are too expensive. 3. Who would probably like to eat at Caf Paris? (A) People who smoke (B) People who like desserts (C) People who don't like to spend a lot of money (D) People who prefer meat Questions 4 through 6 refer to the following article. We asked 100 students, "What is your favorite class?" Here are their answers: Math - 15 Science - 35 English - 20 Music - 30 We also asked the reasons why these students preferred the class. These are the most popular answers: 1) They liked the teacher. 2) The class was easy. 3) They had good friends in the class. We then asked the students, "What is your least favorite class?" Here are their answers: Math - 25 Physical education - 45 History - 15 Geography - 15

39

4. What is the most popular class? (A) Math (B) Science 5. Why might students like their class? (A) Their teacher is angry. (C) They don't like the class.

(C) English

(D) Music

(B) The class is boring. (D) They like their teacher.

6. Which of these classes is most disliked by students? (A) Science (B) Physical education (C) Music (D) English ----------------------------------------------------------------------------------------PART E: SPEAKING TOPIC 2.7: SCHOOL Suggested questions: 1. Did you go to a public high school or a private high school? Why?
........................................................................................................................................................................................................................................................

2. What did you like best about your high school? Why?
........................................................................................................................................................................................................................................................

3. What were some of the rules you had to follow at your high school?
........................................................................................................................................................................................................................................................

4. Which rules did you think were unfair? Why?


........................................................................................................................................................................................................................................................

5. Were you allowed to eat food in the classroom?


........................................................................................................................................................................................................................................................

6. Did you ever get caught breaking any school rules?


........................................................................................................................................................................................................................................................

7. Did you ever skip class? Why or why not?


........................................................................................................................................................................................................................................................

8. Did you belong to any clubs in high school? Why or why not?
........................................................................................................................................................................................................................................................

9. Were you on any sport teams in high school? Why or why not?
........................................................................................................................................................................................................................................................

10. What is your best memory about your high school?


........................................................................................................................................................................................................................................................

40

UNIT 8
PART A: LISTENING

Part 3: Short Conversations Listen and choose the correct answer. 1. What does the man want to do? (A) Use the computer (C) Fix the computer 2. What cant the woman do? (A) Use a computer (C) Sell computers 3. What does the woman have tomorrow? (A) A meeting (C) A business lunch 4. Where are the man and woman? (A) In a library (C) At a train station 5. What does the man want? (A) A ticket to New York (C) A ticket to Los Angeles 6. What is the problem? (A) The man is sick. (C) There are no more tickets.

(B) Break the computer (D) Buy the computer (B) Save documents (D) Find a computer (B) A big project (D) A presentation (B) At the movies (D) At a travel agency (B) A more expensive ticket (D) A train ticket (B) The ticket is too expensive. (D) The man lost his ticket.

Part 4: Short Talks Listen and choose the best answer to each question. 1. What time will the flight now be leaving at? (A) At 6:00 (B) At 7:00 (C) At 7:15 (D) At 7:50 2. At what gate do the passengers now have to leave from? (A) 23A (C) 22B (D) 22A (B) 32A 3. Where is the flight travelling? (A) From Ireland to Spain (B) From Spain to Iceland (C) From Iceland to Spain (D) From Spain to Ireland 4. Where is the bear? (A) To the peoples left (B) To the peoples right (C) Behind the people (D) Above the people 5. Which is closest to Kings age? (A) Five years (B) Seven years (C) Ten years (D) Sixteen years 6. How many bears do they have at Funland? (A) Nine (B) Eleven (C) Seven (D) Ten

41

PART B: VOCABULARY AND GRAMMAR (Relative clauses) Choose the correct word or phrase to complete each sentence. 1. This knife is blunt. It needs to be .. . A. made B. used C. sharpened D. pointed 2. This is Mrs. Edna Campell's kitchen, and on the table there are two of dirty clothes. A. groups B. bands C. piles D. parties 3. When does the ..for your printer expire? A. warranty B. certificate C. license D. approval 4. The employee can seek compensation from the employer for. of contract. A. breach B. fulfillment C. failure D. bleach 5. Lets not.. from the point of issue. A. avoid B. digress C. refuse D. subtract 6. He is determined to from the government. A. stop B. quit C. free D. resign 7. I .for any inconvenience. A. sorry B. regretful C. apologize D. regret 8. James is the one who .the scholarship. A. received B. gifted C. get D. was received 9. he says deserves recording. A. What B. Which C. That D. Who 10. This is the village..he was born . A. when B. how C. from which D. in which 11. Nobody knows for ..Joe is working. A. who B. whom C. that D. which 12. We dont agree with the means ..he solved the problem. A. by it B. by that C. by which D. with it 13 The man you spoke is my uncle. A. to him B. to whom C. whom to D. to that 14. Mary taught me the violin. A. what to play B. where to play C. how playing D. how to play 15. Is this you have been looking for? A. which B. what C. how D. why 16. All ..you have to do is to take care of the baby. A. what B. that C. which D. who 17. Yesterday I met a woman father is an ambassador. A. whos B. who C. whose D. her 18. Not knowing to go, he pulled over to ask directions. A. what B. where C. when D. why 19. Please say .you have on your mind. A. whatever B. whichever C. wherever D. whenever 20. Im going to ask him ..I should do it. A. what B. when C. which D. that

42

PART C: INCOMPLETE TEXTS Choose the best word or phrase for each blank. Questions 1 through 4 refer to the following announcement.

Bournesville Bank is pleased to announce the opening of a new bank at


1109 South Boulevard in the Green Lake section of town. This (1) ------- a full-service branch where you can be sure of receiving the (A) has been (B) will be (C) being (D) was personal, prompt and friendly service that you are accustomed to at all Bournesville Bank branches. Please join us for the grand opening of our new branch on Monday, May 22 during our regular business hours from 9:00 a.m. to 3:00 p.m. There will be refreshments, live (2) -------, and prizes. (A) entertain (B) entertainer (C) entertaining (D) entertainment Bank staff will be on hand to explain all the services available to our customers. Everyone who opens a new (3) ------- during the Grand Opening will receive a (A) vault (B) ledger (C) account (D) entrance special bonus gift. This gift is our way of saying Thank you for doing business with us. Bournesville Bank is your partner in all your business and personal (4) ------- needs. (A) psychological (B) professional (C) educational (D) financial

43

Questions 5 through 8 refer to the following letter.


Rita Harwood Manager Tinkum Square Hotel Portsmouth, NJ 14689

Dear Ms. Harwood: I am writing to let you know of the exemplary care and service I received from (5) ------- staff during my stay at the Tinkum Square Hotel last July. From the (A) my (B) our (C) your (D) their moment of my arrival until the day I (6) -------, I received nothing but courteous and (A) retired (B) resigned (C) departed (D) graduated efficient service from all members of the hotel staff. I especially want to bring to your attention two staff members who provided me with assistance above and beyond the call of duty. On my way to the airport after a pleasant week at Tinkum Square, I discovered that I had left my computer behind. I immediately called the manager on duty, Robert Dunstan, who supervised an emergency search for my computer. After it was discovered in the hotel restaurant, Mr. Dunstans assistant, Martha Jones, got into a taxi and personally delivered the computer to me at the airport. It was all done so (7) ------- that I had my computer in (A) speedy (B) speeded (C) speeding (D) speedily hand well before I had to board the plane. I know that you would like to hear about this example of loyalty and professionalism of Mr. Dunstan and Ms. Jones. I look forward to (8) ------ at Tinkum Square during my next business trip to Portsmouth. (A) staying (B) working (C) looking (D) existing

44

PART D: READING COMPREHENSION Choose the best answer. Questions 1 through 4 refer to the following article. Last night, government officials announced an increase in fares on train service between all major cities in the country. "More and more people are riding the trains these days," said one official. "Trains are a popular form of transportation, but the cost of running them is going up. We need to increase the fares to cover these costs." No changes in train service were announced. Some government officials believe that the increase in train fares could cause more people to travel by bus. The new train fares go into effect at the end of next month. 1. What is the purpose of this article? (A) To report on the popularity of trains (B) To report that train tickets will cost more (C) To compare train travel with bus travel (D) To explain the opinions of government officials 2. When will train fares increase? (A) Next month (B) In a few days (C) Tonight (D) At the end of this month 3. Why will train fares increase? (A) Too many people ride the trains. (B) More train services will be offered. (C) The cost of running trains is increasing. (D) People prefer to take the bus. 4. Why might more people start traveling by bus? (A) Buses are faster. (B) Train tickets will become too expensive. (C) Bus service is better. (D) Trains don't go to many cities.

45

Questions 5 through 8 refer to the following letter and catalog. Dear Mrs. Smith, To thank you for being one of our regular customers, I am pleased to let you know about a great new offer. We are having our end of summer sale. You can find great savings on our catalog prices for T-shirts. We have cut the prices for men's, women's, and children's T-shirts. Your whole family can dress well and look good, but you don't have to spend a lot of money. Please take a look at the attached catalog. Regards, Home and All Ltd.

Men's T-shirt l00% cotton Colors - white, green, blue Sizes - small, medium, large, X-large ............................................ Price - $9.95 Buy 2, get one free Women's T-shirt 100% cotton Colors - white, red, yellow, blue Sizes - small, medium, large ............................................ Price - $8.95 Buy 1, get a free pair of socks Children's T-shirt 100% cotton Colors - red, yellow, blue Sizes - small, medium, large ............................................ Price - $6.95

5. In which colors are men's T-shirts available? (A) White, red, blue (B) White, green, yellow (C) White, green, blue (D) Red, blue, yellow 6. How many men's T-shirts can you get for $40? (A) Three (B) Four (C) Five (D) Six 7. Mr. Smith, Mrs. Smith, and their son all want to buy T-shirts of the same color. What color can they get? (A) White (B) Green (C) Yellow (D) Blue 8. Why did Home and All send Mrs. Smith a catalog? (A) Because she asked for one (B) Because she is a regular customer (C) Because they have a lot of catalogs (D) Because they are friends

46

UNIT 9
PART A: LISTENING Part 1: Picture Description Listen and choose the statement that best describes the picture. 1. (A) (B) (C) (D) 4. (A) (B) (C) (D)

2. (A) (B) (C) (D)

5. (A) (B) (C) (D)

3. (A) (B) (C) (D)

Part 2: Questions and Responses Listen and choose the correct answer. 1. 2. 3. (A) (A) (A) (B) (B) (B) (C) (C) (C)
47

4. 5.

(A) (A)

(B) (B)

(C) (C)

PART B: VOCABULARY AND GRAMMAR (Modification and Word Order) Choose the correct word or phrase to complete each sentence. 1. Its .. hot outside. A. terror B. terrify C. terrible D. terribly 2. They have certainly studied ..to pass the test. A. hard enough B. hardly enough C. enough hard D. enough hardly 3. ..three copies of this letter, please. A. Be B. Do C. Make D. Making 4. If you violate any conditions specified therein, the contract will automatically. . A. complete B. annul C. abolish D. terminate 5. Id like to..my check. A. cash B. charge C. bill D. exchange 6. Can I ..money on line to Brazil? A. refund B. reimburse C. transfer D. repay 7. With her friends ..assistance, she completed her project successfully. A. organ B. organize C. organization D. organizational 8. He scarcely has enough ..to buy new clothes. He often wears old clothes. A. cashes B. coin C. money D. moneys 9. He is a very.person. A. society B. sociable C. sociably D. socially 10. Larry immediately said, I cant go, nor .want to go. A. I B. do C. I do D. do I 11. Mary, to catch up on her rest, went to bed early. A. hope B. hopes C. hoped D. hoping 12. I dont have on the hotels of the island. A. much information B. many informational C. much inform D. many informations 13. In the room, there were more books on biology than expected. A. little B. a little C. much D. many 14. I dont care whether or not. A. his coming B. comes he C. he comes D. will he come 15. They are looking for a .girl named Anna. She got lost in the park. A. seven-year-old B. seven-years-old C. seven-old-year D. seven-old-years 16. I didnt feel well, so I early. A. went back my home B. went my home C. went home D. went to home 17. I dont know whatlike to be famous. A. it is B. is it C. is D. does it 18. He came late yesterday, and so . A. she did B. did she C. she does D. does she 19. .a doctor, Adam rushed over to help the sick child. A. Is B. Was C. Be D. Being 20. I have a feeling that.is going to happen today. A. bad something B. badly something C. something badly D. something bad
48

PART C: INCOMPLETE TEXTS Choose the best word or phrase for each blank. Questions 1 through 4 refer to the following announcement.

The National Museum of Art


is proud to announce the upcoming (1) ------- of European Expressionist (A) exhibit (B) lecture (C) auction (D) purchase paintings and prints, from January 15 through March 15. We are very fortunate to be able to bring this opportunity to area residents and visitors. This show includes works on loan from museums and collectors all over the world. It is the first time this area has seen (2) ------- wide representation of (A) very (B) such a (C) so a (D) enough Expressionist works together in one place. (3) ------- for the show are available by calling the museums Special Events (A) Guides (B) Tickets (C) Procedures (D) Schedules Office at 342-555-0980, or by visiting our website www.artmuseum.org. Prices are $25 general admission and $20 for senior citizens and students with a valid ID. Children under 12 will be charged half price. Entrance prices also include admission to the museums permanent collection. A record tour and headphones will be available at the exhibit for $6. During the show, the Museum Gift Store (4) ----- on sale catalogues, art reproductions, (A) has had (B) had (C) has (D) will have calendars, coffee mugs, and other souvenirs of the show. Questions 5 through 8 refer to the following information. You are never too young or too old to get involved in sports and exercise. In fact the sooner you start to (5) -------, the better. According to some recent research, people (A) exercising (B) sport (C) sporty (D) exercise who started sports at young age were more likely to keep it up as they got older. This applied whether they involved in (6) ------- sports or not. In fact, people who did (A) competition (B) competitive (C) competing (D) compete sports just for fun when they were younger were about 7% more (7) ------- to keep (A) like (B) likeable (C) likely (D) liking exercising when they were older. Researchers believe this might be because some young people get burnt out from competition. However, even people who gave up sport because they felt burnt out tended to return to exercising (8) ------ years later. (A) few (B) a few (C) a little (D) another Therefore, start exercising now if you want to be strong and healthy later.

49

PART D: READING COMPREHENSION Choose the best answer. Questions 1 through 2 refer to the following passage. Owning rental property is a good way to make some extra money, but it is important to manage your property well. The first thing to keep in mind is that you want to keep your property occupied. Every day if your rental apartment or house is vacant, you lose money. This is why it is important to have a good advertising plan. Many landlords hire rental agents to help them find tenants. Others prefer to advertise and show the apartment or house themselves. Whichever way you choose, you want to find a tenant who is responsible and reliable. 1. Who is this passage for? (A) Landlords (C) Rental agents

(B) Tenants (D) Property managers

2. According to this passage, when might a property owner lose money? (A) When he advertises (B) When he hires a rental agent (C) When his property is not occupied (D) When he shows the property himself Questions 3 through 4 refer to the following newspaper article.

All the sports, all the time - World Wide Sports Center. TODAY'S SPORTS SCORES
Baseball Giants 7- 3 Bluebirds Lions 2- 4 Yellow Socks Bears 4- 5 Tigers In baseball today, Bluebirds and Lions fans were upset. The Giants crushed the Bluebirds, and the Yellow Socks beat the Lions. Tiger fans, though, were happy with their team's victory over the Bears. Soccer Tonardoes 0 2 Superstars In soccer news, the Superstars won again - this time beating the Tornadoes. Way to go, Superstars! 3. Which teams won? (A) The Giants, the Lions, the Bears, and the Tornadoes (B) The Bluebirds, the Lions, the Bears, and the Tornadoes (C) The Giants, the Yellow Socks, the Tigers, and the Superstars (D) The Bluebirds, the Yellow Socks, the Tigers, and the Superstars 4. Which team is NOT a baseball team? (A) The Superstars (C) The Tigers (B) The Yellow Socks (D) The Bears

50

UNIT 10
PART A: LISTENING Part 3: Short Conversations Listen and choose the correct answer. 1. Where does the woman want to go? (A) To the library (B) To the post office (C) To the book store (D) Next door 2. How long will it take to walk there? (A) No more than ten minutes (B) Fifteen minutes (C) One hour (D) At least ten minutes 3. When does the woman need to be in the city center? (A) In half an hour (B) In sixty minutes (C) In two hours (D) In forty minutes 4. Where is the man going? (A) To work (B) To school (C) To the store (D) To a funeral 5. What does the woman want? (A) Chinese food (B) A book (C) Some eggs (D) Some milk 6. How many kinds of items will the man bring home? (A) One (B) Two (C) Three (D) Four Part 4: Short Talks Listen and choose the best answer to each question. 1. Why does the man like his job? (A) Its very easy. (B) He works short hours. (C) He likes his co-worker. (D) It does feel like a job to him. 2. What kind of place does the person want to sing in? (A) In a bar (B) In a restaurant (C) In the theater (D) In the park 3. Who does the man sing with? (A) His cousin (C) His brother (D) His friend (B) His father 4. Who is this ad aimed at? (A) People with new cell phones (B) People with old computers (C) People with old cell phones (D) People with new computers 5. When will this sale take place? (A) On Monday (B) On Wednesday (C) On Thursday (D) On the weekend 6. What would you be if you missed this sale? (A) Angry (B) Sad (C) Sleepy (D) Crazy

51

PART B: VOCABULARY AND GRAMMAR (Indefinite pronouns) Choose the correct word or phrase to complete each sentence. 1. Marcel always .. my hair. A. makes B. does C. dress D. takes off 2. A ................ is someone aged between 13 and 19. A. teenager B. youth C. graduate D. student 3. Hemingway was a..of Fitzgerald. A. temporal B. temporary C. temperature D. contemporary 4. A..amount of money has been spent on books. A. considerable B. considerably C. considerate D. considering 5. The United States waged a..war overseas. A. cost-free B. costless C. costly D. cost 6. There is no..evidence that power lines are a health risk. A. final B. last C. conclusive D. finishing 7. He has two dogs: one is from England and is from Germany. A. another B. other C. the other D. the others 8. Most work hard. A. of employees B. employees C. employee D. of employee 9. Sorry, this is taking so long. Can you wait for ..ten minutes? A. other B. some C. another D. few 10. ..in this department was busy preparing for the conference last week. A. Every person B. Every people C. All people D. All the people 11. Dont forget to change the oil .3,000 miles. A. each B. every C. another D. other 12. I dont go to that language school... . A. no longer B. any long C. anymore D. no more 13. Im sure that I would enjoy ..one, the red wine or the white wine. A. others B. both C. either D. some 14. Jane doesnt like her boss, and I dont like mine, . A. either B. neither C. too D. the same 15. The holiday season is by far the busiest time of year for .stores. A. of the most B. most C. the most of D. mostly 16. Dave bought..lamp for his mothers birthday. A. a lovely new B. a new lovely C. a lovelier new D. a new lovelier 17. ..he or she is right. A. Either B. Both C. Each D. Neither 18. Some of the questions were ..that nobody could answer them. A. so hard B. too hard C. very hard D. not hard 19. all of the prisoners were set free. A. Most B. Almost C. Both D. Neither 20. Ive never seen of the two flowers you brought. A. either B. neither C. none D. any

52

PART C: INCOMPLETE TEXTS Choose the best word or phrase for each blank. Questions 1 through 4 refer to the following letter. April 17 To whom it may concern: This letter is (1) ------- for Mr. Young Kim, who has worked for this company as (A) a background (B) an instruction (C) a reference (D) an acceptance an administrative assistant for the past three years. During most of his time here, Mr. Kim has worked directly under (2) ------- supervision. He has served as an (A) your (B) his (C) her (D) my assistant to a busy accounting office with a staff of five. He (3) ------- himself to (A) always has shown (B) always has shows (C) has always shown (D) has shown always be reliable and hard working. He has never shirked his duties, even when the office workload has required him to work late into the evening or on a weekend. I always feel sure that whatever task I may give him, it will be done promptly and with a smile. Mr. Kims friendliness and upbeat attitude have been a real contribution to the office environment. We will miss Mr. Kim, but we understand that he is ready to move on to a (4) ----that will make better use of his skills and provide him with more (A) position (B) degree (C) residence (D) professor opportunities for his future. I can recommend him without reservation and know he will make a great contribution to any work environment. Sincerely, Ivan Sokolow Questions 5 through 8 refer to the following letter. Dear Sarah, Thank you so much for coming to Melody Records to make a demo tape. Here at Melody, we always look for new singers (5) ------ would like to join our record label. (A) which (B) whom (C) who (D) whose Inviting promising singers to come here is the (6) ------- way to listen carefully to a (A) most easy (B) most best (C) easiest (D) most conveniently singers voice. You obviously have a great voice. It was a pleasure to hear you sing. However, after much discussion, we have decided that you do not have the style (7) ------- we are looking for at the moment. Therefore, Im sorry to have to tell y ou (A) who (B) what (C) that (D) when that we are unable to offer you a contract at this time. We will keep our tape in our files. If a suitable project becomes available, we will contact you. You might find it useful to contact (8) ------ record company. (A) other (B) another (C) others (D) the other Sincerely, Rita Branch

53

PART D: READING COMPREHENSION Choose the best answer. Questions 1 through 3 refer to the following e-mail. From: "Hughston, Muriel" <Muriel_Hughston@yahip.org> Date: Mon Dec 1, 2003 4:12:35 PM US/Eastern To: "Steve O'Hara" <s_ohara@verizon.net> Subject: Vacation highlights Dear Steve, I can't believe what a great time I am having here. I have a huge room with a beautiful view of the ocean. At first I had a different room without a view, but then I got lucky. Another guest cancelled his reservation, so I got his room. It's wonderful to have some time off from work. I just relax on the sand all day. I know you don't like the beach, but I wish I could stay here forever. I've reserved a flight home next Sunday. I'm not looking forward to returning to work the next day, but I am looking forward to seeing you again. Love, Muriel 1. Where is Muriel now? (A) At work (B) On an airplane (C) At the beach (D) At home 2. Why does Muriel like her room? (A) It has a view of the ocean. (B) It isn't very big. (C) It is beautiful. (D) It isn't expensive. 3. What will Muriel do on Sunday? (A) She will return to work. (B) She will go to another hotel. (C) She will relax on the sand. (D) She will fly home.

54

Questions 4 through 8 refer to the following directory and e-mail.

Department Store Directory


6 Floor 5th Floor 4th Floor 3rd Floor 2nd Floor 1St Floor
th

Italian Restaurant and Coffee Shop Antique and Modem Furniture Books, Magazines, and Children's Clothing Men's Clothing Women's Clothing Computers and Home Electronics

Today's Value Club Cardholder's Specials: 40% off all his-and-hers cardigans (10:3011:30), 2-for-1 spaghetti lunch set (11:30-13:30), 20% off all Pentium 5 desktop computers (all day). Hours of operation: 9:30 a.m.-10:30 p.m.

To: Club Cardholders <club0223@laceys.co> From: Laceys Department Store Subject: Check out our changes Date: January 30th Dear Club Cardholder, Here at Laceys, we've been trying to make our store even better. We have made a lot of changes and we would like to invite you to take a closer look. We have moved some items, so take a look at the new directory. It will make your shopping easier the next time you visit. Sincerely, M. Morgan Manager, Laceys Department Store 4. Where can you buy a table? (A) On the 2nd floor (C) On the 4th floor 5. What can you buy on the 1st floor? (A) A belt (C) A DVD player (B) On the 3rd floor (D) On the 5th floor (B) A sweater (D) A cup of coffee

6. When can a Value Club Cardholder buy a sweater on sale? (A) In the morning (B) In the afternoon (C) In the evening (D) All day 7. Why did the manager of Laceys send this email? (A) To tell customers about changes (B) To say hello (C) To answer a complaint (D) To tell customers that the store is closing

55

UNIT 11
PART A: LISTENING Part 1: Picture Description Listen and choose the statement that best describes the picture. 1. (A) (B) (C) (D) 4. (A) (B) (C) (D)

2. (A) (B) (C) (D)

5. (A) (B) (C) (D)

3. (A) (B) (C) (D)

Part 2: Questions and Responses Listen and choose the correct answer. 1. 2. 3. (A) (A) (A) (B) (B) (B) (C) (C) (C)
56

4. 5.

(A) (A)

(B) (B)

(C) (C)

PART B: VOCABULARY AND GRAMMAR (Active and Passive) Choose the correct word or phrase to complete each sentence. 1. It is interesting to meet people whose cultural backgrounds ..from our own. A. different B. difference C. differ D. differentiate 2. Its not easy to explain the reason why he has ..to quit. A. choice B. choose C. chose D. chosen 3. He works in the . division of our company. A. personal B. individual C. personnel D. impersonal 4. This is not..business; it can wait. A. tiring B. taxing C. demanding D. urgent 5. Our team is.. to win this time. A. look like B. alike C. likely D. probably 6. Theyve arrangedreception for us. A. heating B. warm C. hospital D. hospitably 7. Left alone, the baby into tears. A. cried B. burst C. poured D. sobbed 8. I cant..it anymore. That website seems to have suddenly vanished. A. reaching B. log out C. access D. pass 9. Last week, John .an award for his community service. A. gave B. was gave C. was given D. had been given 10. I never see him without being.of his grandfather. A. remember B. memory C. remained D. reminded 11. During the experiment, people .to use calculators if necessary. A. were let B. were allowed C. were allowing D. let 12. Im quite sure you will soon grow .to the new work environment. A. accustom B. accustoming C. accustomed D. to be accustomed 13. This mistake .. by him. A. is correcting B. is correct C. has correction D. has been corrected 14. The instructions ..precisely. A. must follow B. must followed C. must to be followed D. must be followed 15. You had better have your teeth at least once a year. A. checked B. to check C. checking D. are checked 16. snow, the mountain looks fantastic. A. Covering by B. To cover by C. Covered with D. To have covered by 17. Everybody the results of the meeting. A. was satisfied with B. satisfied by C. was satisfy D. was satisfying at 18. They were seen kites in the park last weekend. A. fly B. flying C. flown D. flew 19. Once the files are ready, please have them ..me. A. send B. send to C. sent to D. sent into 20. The pictures, by a professional photographer, are going to be on display. A. taken B. which taken C. were taken D. taking

57

PART C: INCOMPLETE TEXTS Choose the best word or phrase for each blank. Questions 1 through 4 refer to the following memo.
To: All personnel From: Marina Petrowki, Director Re: Travel expenses

We are all aware that the procedures for charging and reporting expenses for business trips taken on behalf of the company have long been out of hand. As a result (1) ------- recommendations from the Budget Office, the following (A) of (B) as (C) for (D) on will be adopted. Company personnel will no longer be given credit cards to cover expenses while on out of town trips. Instead, all travel expenses, with the (2) -----(A) excepts (B) excepted (C) exception (D) excepting of airline tickets which will continue to be charged directly to the company, will be paid for out of pocket. In order to receive (3) -------, an expense report must (A) bonuses (B) assistance (C) supervisors (D) reimbursement be submitted to your department head within ten days of returning from a trip. All charges must be itemized on the report and accompanied by receipts. Approval of each item will be made at the discretion of each department head, following, of course, the company expense guidelines (see attached). Generally, charges for hotels, meals, and transportation will be (4) ------. Non-work related items such as (A) reported (B) expensive (C) authorized (D) unallowable entertainment, excessive taxi rides, and bar bill will not. Im sure you will all understand the necessity of this strict attitude toward expense reporting. I am counting on everyones cooperation. Questions 5 through 8 refer to the following warranty. WARRANTY INFORMATION Intosca Sotty warrants the products to be free from defects in materials and workmanship under normal use for a period of one year from the date of purchase. If the product proves to (5) ------ during the warranty period, you should send the (A) defect (B) be defective (C) be defecting (D) defects product to the (6) ------- service center with the proof of purchase date including your (A) finalized (B) essential (C) distributed (D) designated name, address and telephone number. Intosca Sotty will repair or replace the product with a (7) ------- product, and pay for return shipping (8) ------. (A) same (A) charges (B) difference (B) budgets (C) similar (C) fines (D) alike (D) prices

58

PART D: READING COMPREHENSION Choose the best answer. Questions 1 through 3 refer to the following letter.
September 21, 2003 StyleSport Clothing Company 8672 Brooklyn Boulevard West Windsor, VT 00034

To whom it may concern, I am returning a shirt I ordered from your catalog last month. I would like a full refund for this item. When I received it, all the buttons were missing. When I spoke with your customer service representative on the phone yesterday, he informed me that I could not return the item because it was on sale. When I ordered the shirt, however, I did not expect to pay for one without buttons. I am not returning the shirt because of the color or size; I am returning it because it was damaged when I received it. This is the reason I would like my money back. I have been a satisfied customer in the past, and I know you will take care of this matter to my satisfaction. Sincerely, Roger Roberts 1. Why did Roger Roberts write this letter? (A) To complain about the customer service representative (B) To ask for a refund (C) To order a shirt (D) To describe the type of shirt he likes 2. Why doesn't Roger Roberts like the shirt? (A) Its buttons are too big. (B) It isn't a nice color. (C) It is too expensive. (D) It doesn't have any buttons. 3. What does Roger Roberts expect will happen? (A) He will get his money back. (B) The customer service representative will be fired. (C) His complaint will be ignored. (D) The company will send him a new shirt. Questions 4 through 5 refer to the following letter. Dear Grandma, Thank you for the new bike that you gave me for my birthday. I am so happy it is blue, as that is my favorite color. Mom and Dad gave me money and some new clothes, and I got an interesting CD from Uncle Joe. I like your present the most, though. I ride it every day. I promise that I will take good care of it. Love, John 4. What did John's grandmother give him? (A) Money (B) Clothes (C) Roller skates (D) A bicycle 5. What does John promise to do? (A) He promises to ride the bicycle every day. (B) He promises to take care of the bicycle. (C) He promises to spend his money wisely. (D) He promises to write to his grandmother every day.

59

UNIT 12
PART A: LISTENING Part 3: Short Conversations Listen and choose the correct answer. 1. Where is this conversation taking place? (A) On the telephone (B) On the radio (C) On the television (D) In a store 2. What does the man want to know? (A) The womans phone number (B) How to find the book store (C) When the store is open (D) The price of a book 3. What time does the store open on Saturday? (A) Ten oclock (B) Nine oclock (C) Five oclock (D) Six oclock 4. What color are the gloves the man is searching for? (A) Red (B) Brown (C) Blue and brown (D) Blue 5. Who gave the gloves to the man? (A) His wife (B) The woman (C) His mother (D) His sister 6. What does the woman say about the gloves? (A) She wants a pair like them. (B) She has a pair like them. (C) They are on the chair. (D) She doesnt know where they are. Part 4: Short Talks Listen and choose the best answer to each question. 1. How did the speaker feel when she first went to America? (A) Lonely (B) Happy (C) Sad (D) Hungry 2. Which of these did the speaker NOT mention about first being in America? (A) She didnt like the food. (B) She had no friends. (C) She didnt like the weather. (D) She didnt have any fun places to go to. 3. Why does the speaker like America now? (A) She likes the weather and the food. (B) She has lots of friends and likes the weather. (C) She has lots of friends and things to do. (D) She likes the weather and has lots of things to do. 4. When did the speaker start taking Taekwondo lessons? (A) Last month (B) Two years ago (C) Last year (D) Last Christmas 5. How does the speaker feel after doing Taekwondo for so long? (A) Faster and angrier (B) Stronger and lighter (C) Lighter and angrier (D) Faster and stronger 6. What does the speaker want to get next year? (A) A red belt (B) A blue belt (C) A brown belt (D) None of the above

60

PART B: VOCABULARY AND GRAMMAR (Conjunctions and Prepositions) Choose the correct word or phrase to complete each sentence. 1. He left home early...failed to catch the train. A. and B. therefore C. but D. or 2. ..he was cooking, his wife was working in the yard. A. When B. While C. As soon as D. Since 3. He works to support his family..he is in his seventies. A. in spite of B. despite C. though D. because 4. I havent seen him ..ages. A. for B. with C. during D. until 5. He didnt failed the test; .., he got the highest score. A. likewise B. else C. otherwise D. on the contrary 6. after his arrival in Paris, Tom rang me. A. Briefly B. Shortly C. Nearly D. Simply 7. Not only does Steve sing at a bar,..he also serves the patrons. A. and B. or C. but D. still 8. ..I was waiting in line for a visa interview, I ate my lunch. A. Although B. While C. Since D. Because 9. He wouldnt say anything ..directly asked a question; he was a man of few words. A. since B. otherwise C. or else D. unless 10. .Im retired, I can take a long journey throughout Europe . A. However B. Now that C. Thanks to D. Due to 11. I drive to school ..my brother always cycles. A. when B. even C. whereas D. if 12. It was not until the 1950s ..an effective vaccine for that virus was found. A. then B. when C. that D. which 13. I have go to the meeting..I want to or not. A. because B. whether C. as soon as D. while 14. Youd better take an umbrella ..it may rain. A. as soon as B. despite C. because D. although 15. You can take a rest as long as you want ...you finish the report by 5 p.m. A. so that B. although C. provided that D. while 16. The manager was preoccupied .checking the report. A. in B. at C. with D. from 17. She was standing by the window..her eyes closed, listening to music. A. while B. with C. about D. except 18. I have no objection, if everyone else agrees, leading the meeting. A. to your B. for your C. to youre D. on your 19. Just ..you and me, I prefer the restaurant where we ate last Friday. A. among B. around C. between D. from 20. She doesnt want to fail the test again; .., she has been trying her best to study all the time. A. however B. consequently C. nonetheless D. on the other hand

61

PART C: INCOMPLETE TEXTS Choose the best word or phrase for each blank. Questions 1 through 4 refer to the following information.

Volunteer Wanted
The Psychology Department of Queenstown University is (1) ------- volunteers (A) searched (B) looking for (C) watching (D) investigating to take part in an experiment. We need 150 volunteers: 75 men and 75 women. Volunteers should be aged (2) ------- 20 and 25 and should not be taking any kind (A) under (B) from (C) between (D) by of medication. Volunteers must not be enrolled in any of the university current psychology courses. The experiment will take from thirty to forty minutes to complete. Participants will be asked to answer a series of questions after (3) -----(A) have watched (B) watch (C) watching (D) being watched a short film. A small fee will be paid to participants. Coffee and tea will be also served during the film. If you are (4) ------, please contact the Psychology (A) interested (B) interesting (C) interested in (D) interest th Department secretary on 900-8887 before January 30 . Questions 5 through 8 refer to the following letter.
Huntley school for girls Huntley H78 E88

Dear Mrs. Miles, It is with great (5) ------- that I am writing this letter. As you know, your daughter (A) pleasure (B) joy (C) regret (D) upset Miranda has been having problems with some of the other girls. Unfortunately, on Monday, there was an unforgiveable incident (6) ------- a math lesson. One girl (A) while (B) until (C) during (D) when needed to be hospitalized as a result. We cannot tolerate this kind of behavior. Miranda has not listened to any of our warnings and I need to talk to you (7) -------. We are considering (8) ------ Miranda, but would like to talk to you first. (A) in person (A) to expel (B) with people (B) for expelling (C) in people (C) the expel (D) as person (D) expelling On the other hand, her sister Melinda is a delight to teach. We hope that she will continue her studies at Huntley. Sincerely, Marjorie Bartle, Principal

62

PART D: READING COMPREHENSION Choose the best answer. Questions 1 through 4 refer to the following letter.
Dear Agnes, I'm so g1ad I come to Mexico for my vacation. The food here is quite tasty and there is fresh fruit everywhere. I have tried several of the local specialties. A lot of the food is too spicy for me, but they also serve some mild dishes. Some of the dishes are fried, which I dont like, but they also have other ways of preparing the food. I had a wonderful bak ed fish last night. And there is freshly baked bread every morning. You probably think that all Ive done on this trip is eating. Youre right! Its been cold and rainy most of the time, but I dont mind as long as I can enjoy a warm meal and a cup of coffee in a Mexican caf. I hope youll come here with me next year. Youll love it. Love, Hilda 1. Why did Hilda write this letter? (A) To tell Agnes about her vacation (B) To tell Agnes about her favorite restaurant (C) To explain to Agnes how to cook Mexican food (D) To explain to Agnes what her favorite food is 2. What kind of food does Hilda prefer? (A) Spicy (B) Fried (C) Baked (D) Cold 3. What does Hilda suggest to Agnes? (A) To go to a Mexican restaurant (B) To visit Mexico next year (C) To learn how to cook Mexican food (D) To have a cup of coffee 4. What does Hilda think of the weather? (A) She doesn't mind it. (B) It's very warm. (C) She loves it. (D) It should rain more.

Questions 5 through 8 refer to the following notice and text message. We are sorry to report that Mr. Smith is sick today. Because of Mr. Smith's illness, his classes scheduled for today will be canceled: Beginning English, Advanced English, English Listening, and English Reading. Mr. Smith and the English department apologize for any inconvenience Mr. Smith's absence creates. Mr. Smith will be back tomorrow. Each student is responsible for reading the next chapter in his or her textbook. Please join us in wishing Mr. Smith a quick recovery.
Hey Jo! Did you see the notice? Mr. Smith is sick, so we've no class today. I'm so happy. I forgot to do my homework. Now I have an extra day. Let's talk about Jessie's party during class time. I don't want to read a boring book. Send me a reply! Sally 5. How many classes does Mr. Smith normally teach? (A) Two (B) Three (C) Four (D) Five 6. When will Mr. Smith be back? (A) Today (B) Tomorrow (C) Next Friday (D) Next week 7. What should Mr. Smith's students do? (A) They should come back next week. (B) They should read the newspaper. (C) They should buy their textbooks. (D) They should read a chapter. 8. What will Sally probably do during class today? (A) Write an essay (B) Chat with her friend (C) Read a book (D) Take a nap
63

UNIT 13
PART A: LISTENING Part 1: Picture Description Listen and choose the statement that best describes the picture. 1. (A) (B) (C) (D) 4. (A) (B) (C) (D)

2. (A) (B) (C) (D)

5. (A) (B) (C) (D)

3. (A) (B) (C) (D)

Part 2: Questions and Responses Listen and choose the correct answer. 1. 2. 3. (A) (A) (A) (B) (B) (B) (C) (C) (C) 4. 5. (A) (A) (B) (B) (C) (C)

64

PART B: VOCABULARY AND GRAMMAR (Review) Choose the correct word or phrase to complete each sentence. 1. Can you finish your homework ..next Thursday? A. then B. by C. until D. till 2. Nowadays, most ..have four wheels. A. cars B. of the cars C. the cars D. among the cars 3. There are .more books that are beneficial to you. A. so B. many C. such D. much 4. He .that smoking indoors not be allowed under any circumstances. A. refuses B. insists C. conditions D. regulates 5. This program is .harder to get accustomed to than the old one. A. such B. so many C. very D. much 6. When he got home, his wife ..dinner. A. fix B. fixes C. fixing D. was fixing 7. The Joneses must have left at least one week. A. since B. ago C. advanced D. during 8. There was left in the safe. A. two-hundred dollar bills B. two-hundreds dollar bills C. two hundred dollars D. two-hundreds dollars 9. at a distance, it looks like a miniature city. A. seen B. seeing C. having seen D. having been seen 10. This math problem .solved in ten minutes. A. will able to be B. be able to is C. is able D. can be 11. Though they look very much alike, the two brothers have personalities. A. different B. difficult C. varied D. differentiated 12. The new employee is neither ambitioushard-working. A. or B. nor C. and D. as well as 13. He wouldnt leave the platform the train went out of sight. A. so that B. until C. so D. therefore 14. We had ..stay out too late. A. not rather B. rather C. not better D. better not 15. Brian was seen .his car. A. washing B. wash C. have washed D. washed 16. For more.., contact us at 721-3431. A. informations B. informed C. further information D. information 17. Carry an umbrella ..in case it should rain. A. with you B. by you C. with yourself D. by yourself 18. A customer asked to see the manager .. . A. in face B. in front C. in person D. indirect 19. The woman in black comes from a ..family. A. respecting B. respects C. respectable D. respective 20. This is the third time that the lenses of my glasses have ..broken. A. all B. both C. each D. any

65

PART C: INCOMPLETE TEXTS Choose the best word or phrase for each blank. Questions 1 through 4 refer to the following e-mail.
To: All employees From: Henry Morrison, Vice President Subject: Everybody Parking

I have been told that there are concerns about the parking spaces. Thus, the executives have finally decided (1) ------- the current employee parking lot. (A) expansion (B) for expanding (C) to expand (D) expand (2) -------, we have agreed to compensate employees for public transportation (A) On the contrary (B) Therefore (C) Furthermore (D) Despite fees should they choose to (3) ------ buses or train to work. The decision (4) -----(A) take (A) on the amount of (B) go (B) for the number of (C) get on (C) on the number of (D) travel (D) to the amount of compensation will be made very soon. This should also prompt our employees to utilize more public transportation, thereby ensuring adequate parking spaces. Regards, Henry Morrison, VP Questions 5 through 8 refer to the following notice. Attention Museum Patrons We would like to give you advance notice about a (5) ------ of the National Museum. (A) temporarily close (B) temporarily closing (C) temporary closure (D) close temporary There (6) ------- a major ceremony for veterans of World War II in the main gallery (A) was (B) will be (C) would have been (D) has been at dawn on Friday, April 17. Admission to the event is by invitation, with spaces reserved for public figures and veterans and families only. (7) -------, the museum will be closed to the general public for the entire morning. (A) Accord (B) Accordance (C) According (D) Accordingly The museum will not reopen for normal business until 12:30 p.m. in order to give the staff time to clean up after the (8) -------. (A) function (B) issue (C) instance (D) situation We would like to apologize to our regular patrons for the inconvenience. Thank you for your understanding in this matter.

66

PART D: READING COMPREHENSION Choose the best answer. Questions 1 through 3 refer to the following memo. To: All staff members From: Personnel Director Re: Company health benefits All company employees are now allowed 12 days of sick leave a year. Remember, these days are to be used in case of illness and not as vacation days. In addition, starting next month, company employees can receive a 15% discount off all prescription medicine bought at the Center Pharmacy on Elm Street. Just show your company card to the pharmacist when ordering your medicine. 1. What is the purpose of this memo? (A) To explain where the Center Pharmacy is (B) To announce new health benefits (C) To explain the difference between sick leave and vacation days (D) To discuss the cost of prescription medicine 2. How many vacation days a year are company employees allowed? (A) 12 days (B) 15 days (C) One month (D) It isn't mentioned 3. What will happen starting next month? (A) All employees will get a vacation. (B) Employees will be able to get a discount on prescriptions. (C) Employees will be allowed 12 sick days a year. (D) All employees will be given a company card. Questions 4 through 6 refer to the following advertisement.

Always tired? No energy?


Do you need a little extra strength to get through that tough workday? If you answered "yes," then try Vitagetic, the new energy drink. Vitagetic has all the vitamins that your body needs to maintain a high level of energy all day long. It is an easy way to get all your vitamins. You don't have to take lots of vitamin pills. It comes in three fantastic flavors: Lemon, Orange, and Apple, and tastes great. Have one bottle at breakfast and you will feel healthy and energetic until quitting time. You can find it at all good supermarkets. 4. Who should drink Vitagetic? (A) People who are often tired (B) People who have a lot of energy (C) People who are bored (D) People who can't sleep 5. Why is Vitagetic good for you? (A) Because it has oranges and apples (B) Because it has vitamins (C) Because it comes in a bottle (D) Because you drink it at breakfast 6. Which of the following is NOT true? (A) You have to take vitamin pills with Vitagetic. (B) You can buy Vitagetic at supermarkets. (C) Vitagetic is a good breakfast drink. (D) Vitagetic is delicious.

67

UNIT 14
PART A: LISTENING Part 3: Short Conversations Listen and choose the correct answer. 1. Who is Mary? (A) The new accountant (B) The old accountant (C) A new teacher (D) The manager 2. What is Mary wearing? (A) A black dress and red jacket (B) Red pants (C) A black suit (D) A red dress and a black jacket 3. Why is the man surprised? (A) He thought Mary was taller. (B) He expected Mary to talk to him. (C) He thought Mary would look older. (D) Mary is his old school friend. 4. What are the man and woman talking about? (A) Making cookies (B) Going on a picnic (C) Open a restaurant (D) Going on vacation 5. What does the woman want to know? (A) If she needs to bring anything (B) If she should pay a fee (C) If the man can give her a ride (D) What time she should arrive 6. When will the man and woman next meet? (A) The following morning (B) Tomorrow night (C) Next week (D) At 11 p.m. Part 4: Short Talks Listen and choose the best answer to each question. 1. What does Pablo enjoy about Korea? (A) The universities (B) The food (C) The economy (D) The cities 2. What is Pablo studying? (A) Spanish and business (B) Korea and cooking (C) Spanish and Korea (D) Economics and Korea 3. Where is he going next and when? (A) Madrid, next January (B) Barcelona, next January (C) Barcelona, next February (D) Madrid, next February 4. What is the problem? (A) A car has a flat tire. (B) A car has been stolen. (C) A car has broken down. (D) A car is blocking the entrance. 5. What is the registration number of the car? (A) ST 4572 (B) ST 4571 (C) SP 4571 (D) SP 5571 6. What should the owner of the car do? (A) Change the tire (B) Go to the parking lot (C) Call the police (D) Buy a new car

68

PART B: VOCABULARY AND GRAMMAR (Review) Choose the correct word or phrase to complete each sentence. 1. My umbrella by that terrible wind this morning. A. broke B. was broken C. break D. had broken 2. He wanted the exam. A. to not fail B. not to fail C. to fail not D. failing not 3. . the research uncovered some problems, the company decided to stop the project. A. Since B. Wherever C. Despite D. Even though 4. They say that the has not yet been decided. A. deadline B. finishing line C. clothes line D. underline 5. With only one more week, he ..better prepared for the concert last Saturday. A. could have been B. could be C. could well have D. could well be 6. The photocopier needs . A. to fix B. to be fix C. fixing D. to be fixing 7. Brian and Diana phone ..almost every day. A. themselves B. one another C. each other D. the other 8. Lets take another .at the sales figure. A. view B. scene C. glance D. vision 9. It is reported that the boat..about 60 miles off the coast of South Africa. A. disappeared B. was disappeared C. was being disappeared D. has been disappeared 10. I am considering . A. taking a new job B. to take a new job C. take a new job D. taking new job 11. What will become ..the child now that his parents are gone? A. with B. at C. of D. to 12. The instructor told us ..to find the book. A. whom B. which C. where D. what 13. There are few mothers ..dont love their own children. A. who B. which C. whom D. except 14. More and more overseas visitors are coming to this city clothes. A. bought B. with buying C. in order that buy D. to buy 15. Who is that man..sunglasses? A. wears B. is wearing C. wear D. wearing 16. .in a lake is very different from swimming in an ocean. A. to swim B. swimming C. to be swimming D. swim 17. There are no .between the brothers. A. same B. difference C. similarities D. variation 18. In addition to English, my daughter can French. A. talk B. say C. speak D. tell 19. On my trip to Italy, not only a suitcase, but I broke my glasses as well. A. I lost B. I lose C. did I loss D. did I lose 20. Since the early 1990s, the demand for personal computers. . A. steadily increases B. steadily increased C. has been steadily increasing D. have steadily increased
69

PART C: INCOMPLETE TEXTS Choose the best word or phrase for each blank. Questions 1 through 4 refer to the following notice. Please Note! Many employees have been complaining recently that the refreshment area is often left untidy. It is in everyones interest that this space should remain (1) ------- , so please do your best to keep it that way. (2) ------- instance, after (A) cleanly and order (A) For (B) clean and order (B) In (C) clean and orderly (C) On (D) cleanly and in order (D) To you use the cups or dishes, do your best to clean them and put them away. To make sure that we do not have any further problems, I will introduce an employee roster system for kitchen area duties. This means that you will have to take (3) ------ looking after the area. Duties will include making sure the bench (A) turns (B) rides (C) tries (D) efforts is clean, restocking the coffee and milk, and washing any leftover dishes. I will send you each a copy of the roster (4) ------ I have finished drawing it up. (A) during (B) when (C) while (D) before Thanks, Jenny Reid, Office Manager Questions 5 through 8 refer to the following e-mail.
From: david@cereamsonbooks.com To: Aaron@gmail.com Date: September 29 Subject: Your order

We regret (5) ------- you that the book you are interested in purchasing is currently (A) to notice (B) to notify (C) noting (D) notifying out of stock. However, we were informed by the publishing company that it would be available (6) ------- the next two weeks. (A) until (B) between (C) within (D) since If this (7) ----- is still acceptable to you, please let us know by return e-mail (8) ----(A) moment (A) in order to (B) author (B) so that (C) advantage (C) so as (D) option (D) for we can order the item for you. We look forward to hearing from you soon. Thank you for placing an order with us. Regards, David Martin Creamson Books

70

PART D: READING COMPREHENSION Choose the best answer. Questions 1 through 3 refer to the following memo. To: All teachers From: Wilma Harris, principal Re: Attendance Forms It has come to my attention that many of you have been turning in your attendance forms late. Please remember that the deadline for submitting attendance forms is 3:30 p.m. on the last school day of every month. Also, you must call the parents of every child who is absent three or more times without an excuse. Thank you. 1. Why did Wilma Harris write this memo? (A) Too many students have been absent from school. (B) Teachers have been submitting their attendance forms late. (C) Students haven't been paying attention in class. (D) Teachers have been arriving at school late. 2. How often must teachers turn in their attendance forms? (A) Every day (B) Twice a month (C) Once a month (D) Once a year 3. When should teachers call a childs parents? (A) Every time a child is absent (B) When a child doesn't have an attendance form (C) When a child misses school three times (D) Every time a child is late Questions 4 through 7 refer to the following letter. April 10th, Dear Harry, Our new house is so nice. We have been living here for two months. We have such a big living room and dining room. Upstairs, there are three bedrooms and two bathrooms. Behind the house, we have trees, a swimming pool, and a garden for growing vegetables. In the front, we have a beautiful flower garden. I hope that you will come to visit someday. Best wishes, Jack 4. What is Jack writing about? (A) His new house (B) His family 5. What is upstairs? (A) The living room (C) The swimming pool 6. What is in front of the house? (A) A vegetable garden (C) A swimming pool 7. When did Jack move to his new house? (A) In two months (B) Recently (C) His dog (B) The kitchen (D) Three bedrooms (B) A flower garden (D) Three bathrooms (C) In April (D) In February (D) His friend

71

LANGUAGE FOCUS
AUXILIARY VERBS
1. What Follows Auxiliary Verbs? A. Auxiliary verbs are followed by a verb ending in ing or ed. The boy is flying a kite. The book was listed as a best seller. The boy has closed the window. B. Auxiliary verbs are followed by the basic form of the verb. Modal auxiliaries do in question/negative sentences, or emphatic do + infinitive Paul may arrive tomorrow. I do believe that he is right. 2. Meaning of Key Auxiliary Verbs A. Will: be willing to; intend to I will open the door for you. Where shall we sit? C. May / Can: be possible to do Can fish live in the river? D. Can / Could / May: be allowed to do May I have one? Note: negative = cannot/cant, must not This step must be next. F. Must / Have to: be required to do Everyone must be on time. You should call her. She should have read the book. I. Used to / Would: did in the past

Do you like fish? He does not like playing cards.

I will be there at 8:00. We shall ask the committee. It may rain tomorrow. You can take both of them.

B. Shall: intend to (formal); have decided to

E. Must: be necessary; be logically certain It must be her purse. They have to work late. We ought to get there early. They could have won, but they didnt. In the spring, I would plant flowers.

G. Should / Ought to / Had better: be obliged to do

H. Should have / Could have + p.p.: was not true; did not happen

We used to talk every day.

72

ENGLISH VERB TENSES


1. Present: Simple, Perfect, and Progressive A. Use present simple for routines, habits, or thing that are always true. He takes the subway. (every day) B. Use present progressive (is/are ing) for a temporary routine or situation He is taking the subway. (now) C. Use present perfect (have/has ed) for a situation where things that happened in the past have a result in the present. Note: Present perfect often goes with just, yet, for, since, ever, never, first time, second time, this week. He has just cut himself. ( We can see the direct immediate result of this action the blood.) 2. Past: Simple, Perfect, and Progressive A. Use past simple for something that was finished in the past. She traveled in Europe. (at some time in the past) B. Use past progressive (was/were ing) for a temporary routine or situation that was in progress at a specific time in the past. She was traveling in Europe when I last saw her. C. Use past perfect (had ed) for something that happened before a certain point in time in the past. She had traveled in Europe before she decided to study there. 3. Future: Use of Will and Other Words with Future Meaning A. Use will for instant decisions. OK. I will accept your offer. B. Use be going to for established plans. They are going to buy drinks for the party. C. Use present simple instead of future tense in adverbial clauses led by if, the moment, when, etc. We will start the moment he arrives.

73

INFINITIVES AND GERUNDS 1. Infinitives


A. Use a to-infinitive after the following verbs: agree, decide, expect, happen, pretend, promise, manage, tend (for example, decide to do) B. Use a to-infinitive after the following verbs plus their object: advise, allow, expect, forbid, want, force, tell (for example, allow him to do) C. Use an infinitive without to after the following verbs plus their object: have, let, see, make, feel, see, hear, smell, find (for example, let Tom do it) 2. Gerunds Use an ing form after the following verbs: avoid, cant help, deny, feel like, give up, imagine, mind, postpone, enjoy (for example, deny giving up) 3. Choosing Between Infinitives and Gerunds The following verbs allow both a to-infinitive form and an ing form: remember, forget, stop, regret, try. The choice of a to-infinitive form and an ing form depends on the meaning. I forgot to turn the light off. (= I didnt turn it off. I forgot to do it.) I forgot turning the light off. (= I actually turned it off. I forgot that I had done that.)

4. Choosing Subjects A. Use objective case pronouns with a to-infinitive. expect him to help her enjoy their singing allow them to do it mind my smoking

B. Use a possessive adjective with a gerund.

74

PARTICIPLES AND PARTICIPLE CLAUSES


1. Forms of Participles: -ing and ed/V3 A. If the participle of a noun is the agent (doing the action of the participle), choose an ing form. The game tired the players. The tiring players (agent) B. If the participle is NOT the agent, choose an ed form. The game tired the players. The tired players (not agent)

2. Participle Clauses A. An ing form can be used when two things occur at the same time. He suddenly went out shouting. B. An ing form can be used when one action occurs during another action. She hurt herself cooking dinner. C. An ing form clause can be an explanation of the following main clause. Feeling tired, he went to bed earlier. 3. Choice Between ing and ed in Participle Clauses A. Use an ing form when the original verb is put in the active voice. Walking along the street, Tom ran into one of his old friends. B. Use an ed form when the original verb is put in the passive voice.

Located on a hill, the hotel overlooks a nice lake.

75

NEGATION AND PARALLEL STRUCTURE


1. No vs. Not A. Use no as an adjective. no time B. Use not as an adverb. not going not busy not likely no choice

2. Main and Auxiliary Verbs A. Put not or never after an auxiliary verb. He will not arrive today. She does not know him. You should never eat there. They do not live in Chicago.

B. If there is no auxiliary, do-support is necessary along with not.

3. Non-finite Verbs: To-infinitives, Gerunds, and Participles Put a negative word before non-finite verbs. He prefers not taking the bus to work. In order not to fail again, you should work harder. Not knowing what to do, he just stood still. 4. Redundancy in Negation A. Avoid double negatives. She does not have no money. (incorrect) She does not have any money. (correct) B. Do not use a negative word along with such words as hardly, seldom, scarcely, lest, unless. We could not hardly understand the situation. (incorrect) We could hardly understand the situation. (correct) 5. Parallel Structure A. Parallelism between two or more words He is handsome, honest, and has a lot of money. ( rich) B. Parallelism between two or more phrases

You may answer the question either in English or Korean. ( in Korean)

76

COMPARISONS
1. Regular comparison A. When comparing things, shorts adjectives end in er/-est. dark darker darkest hot hotter hottest happy happier happiest scary scarier scariest B. For long adjectives (three or more syllables) used in comparisons, place more, most, less, or least before the adjective. beautiful more beautiful most beautiful expensive less expensive least expensive 2. Irregular Comparison A few adjectives and adverbs have comparative and superlative forms that are different from their regular forms. good/well better best much/many more most bad/badly worse worst little less least

far farther/further farthest/furthest Note: Further is used for both greater distance and more of something, while farther is only used for greater distance. 3. As As comparatives Only the regular form of an adjective or adverb comes between as and as; for example, He is as old as me. He speaks English as fluently as his brother does. 4. Modification of Comparatives Comparatives can be modified by the following words: much, very much, far, a little, a bit, a lot, any, no, even, still. He is a bit taller than me. This is even worse than that one. 5. Choose Between Comparatives and Superlatives A. Choose a comparative form if than appears in the sentence. He is taller than his brother. B. The presence of ever, (of) all, possible, in the world is a strong suggestion that a superlative is needed. This is the highest building I have ever seen.

77

AGREEMENT
1. Subject-Verb Agreement A. Modifiers do not affect the number of the verb. The teacher, along with his students, wants to play soccer. (The phrase along with his students is a modifier.) B. Expressions of time, distance, price, and weight need a singular verb. Ten dollars is too much for a lunch. C. In case of fractions or their equivalents, the noun in the of-phrase determines the number of the verb. Two-thirds of the land has been sold. Two-thirds of the students in that class are male. D. The number of the verb is determined by what is next to it in sentences with either A or B, or neither A nor B. Either he or his pupils are going to help us.

2. Adjective/Determiner-Noun Agreement Modifiers and nouns modified by them agree in number. this/that book every girl another boy each car these/those books all girls several boys most cars

Note: Every and another may be followed by either a singular or a plural noun in expressions regarding time or distance. every day/mile every two days/miles another week/mile another two weeks/miles 3. Noun-Pronoun Agreement Pronouns including reflexives must agree with their antecedent in number. Johns family are all early risers; they (NOT it) rise at 6 a.m. They advised her to take care of herself (NOT themselves).

78

RELATIVE CLAUSES
1. Relative Pronouns A. Who, which, that Who is used when the antecedent is a person, while which is used when the antecedent is not a person. However, that as a relative pronoun can replace either who or which, and it is preferred especially after all, everything, nothing, the only, and superlatives. The woman is a singer. + She lives next door. = The woman who/that lives next door is a singer. B. Notice that which can refer to the entire preceding clause. John didnt pass the test, which (NOT that or what) disappointed his father. C. Possessive of the relative pronouns Possessive can be expressed by whose, whether or not the antecedent is a person. When the antecedent is not a person, of whichmay also be used. The man is my uncle. + His car broke down. = The man whose car broke down is my uncle. D. What (= the thing which) The relative pronoun what is different from the other relative pronouns in that it is not preceded by a noun. He believes what he hears. (= He believes the thing that/which he hears.) 2. Relative Adverbs When, Where, How, Why the day/month/year/time etc. + when: I remember the day (when) we met. the hotel/park/place etc. + in which OR where: Is this the park in which you two met? Is this the park where you two met? the way + how: This is the way he did it. This is how he did it. Note: The way and how cannot be used together. the reason + why: This is the reason (why) he got upset.
79

MODIFICATION AND WORD ORDER


1. Modification A. Adverbs modify adjectives, adverbs, and verbs. He is an extremely rapid thinker. (NOT extreme rapid) She walks elegantly. (NOT walks elegant) It grows extremely slowly. (NOT extreme slowly) cf. That sounds interesting. (NOT sounds interestingly) B. Adjectives modify nouns. No more Canadian products are available here. (NOT Canada products) 2. Word Order A. Enough comes after adjectives/ adverbs and before nouns. He is old enough to see that movie. (NOT enough old) We have enough books on this topic. (NOT books enough) B. Words ending in thing, -body, -one, -where come before their adjectival modifiers. I have something nice. (NOT nice something) I have gone somewhere new. (NOT new somewhere) C. Subject-verb inversion Questions involve inversion between subject and verb, but indirect questions involve no such inversion. I asked him how long he had been in America. (NOT how long had he been) When the sentence begins with a negative word, the subject comes after an auxiliary verb. Rarely does he work at night. (NOT Rarely he works) Not until yesterday did he change his mind.

80

INDEFINITE PRONOUNS
Example of Indefinite Pronouns and Their Usage A. Examples of Indefinite Pronouns all, both, every, everything, each, some, somebody, someone, something, any, anybody, anyone, anything, none, either, neither, one, other, nothing, most of B. Both, Either, Neither Both: Means two (things/people) and is often followed by of + plural noun or and Either: Means one or the other of the two (things/people) and is followed by of + plural noun or or. Neither: Means not either one of the two (things/people) and is followed by of + plural noun or nor. C. Some, any Some: Used in affirmative sentences. Some can be used in questions when an affirmative answer is expected. Any: Used in questions or in negative sentences. D. One, Another, One / The other: Used when there are two things/people, etc. Of the two pens, one is red and the other is white. One / Another / The other: Used when there are three things/people, etc. He has three brothers: one is a doctor, another is a professor, and the other is a singer. One / Another / The others: Used when there are more than three things/people, etc. We have five dogs: one is from Scotland, another is from Germany, and the others are from Spain. E. Most of vs. Most Use most of to talk about particular things or people, and use most to talk about things or people in general. Most of the students in this class are freshmen. Most people love music.

81

ACTIVE VOICE AND PASSIVE VOICE


1. Active vs. Passive Sentences A. Formula Active: Subject Verb Object

Passive: Subject Be PP Preposition Object B. Use of by + noun With passive, we can use by + noun if we need to say who does the action. The police caught him. He was caught by the police. 2. General Key Points A. Only transitive verbs may be changed to passive forms. The ship has been disappeared. (incorrect) English is spoken here. (correct) B. Verbs like make, tell, and ask are followed by a to-infinitive in passive sentences. The boys were made to work very hard. The girls were asked to stop laughing. C. Perceptual verbs like see, observe, and hear are followed by either a to-infinitive (for repeated or habitual actions) or an ing form (for an action in progress) in passive sentences. The geese were observed to fly south every autumn. (habitual) The geese were seen swimming in the pond. (action in progress) D. Complex verbs that consist of verb + preposition are treated as one unit when changed to passive. They laughed at him. He was laughed at (by them). Nobody has slept in this room. This room has not been slept in (by anybody). E. Besides, the prepositions at and with may also be used in passive sentences. At: be alarmed at, be surprised at With: be pleased with, be satisfied with, be covered with, be (pre)occupied with At or With: be delighted at/with, be disgusted at/with

82

CONJUNCTIONS AND PREPOSITIONS


1. Coordinating Conjunctions FANBOYS (for, and, nor, but, or, yet, so) For (=because): Im not going to join the club, for Im not sure of its purpose. Yet/But (=however): He has a good job, yet he never seems satisfied. Nor (=and not): Dick didnt pass the test, nor did his friend, John. So (=therefore): My foot still hurt yesterday, so I went to see a doctor. 2. Subordinating Conjunctions Simultaneous actions: while, as, the moment/instant Sequential action: when, before, after, until Reason: since, because, now that, seeing that Condition: if, unless, provided, as long as, once Concession: even if, even though, although, though Purpose: (so) that, in order that Contrast: whereas, while Unreal Comparison: as if, as though 3. Distinction Between Conjunctions and Prepositions Conjunctions are followed by subject + verb, while prepositions are followed by noun or an ing form. while vs. during (al)though vs. despite because vs. because of 4. Key Prepositions A. for (how long) vs. during (when) for two weeks; during the winter B. by (completion) vs. until (continuation) have it done by tomorrow. Use the car until this evening. C. between (two) vs. among (more than two) between the eyes; among the crowd D. Prepositions at the end of questions Who did you come with? What is this for? What is it like?

83

REFERENCES
Lee, J. (2009). TOEIC Academy. Ho Chi Minh General Publish House. Lougheed, L. (2009). Preparation Series for the New TOEIC Test: Introductory Course (4th edition). Longman. Lougheed, L. (2009). Preparation Series for the New TOEIC Test: Intermediate Course (4th edition). Longman. Taylor, A. & Byrne, G. (2008). Very Easy TOEIC (2nd Edition). First News. Taylor, A. & Malarcher, C. (2007). Starter TOEIC (3rd Edition). First News. Trew, G. (2007). Tactics for TOEIC. Oxford: Oxford University Press.

84

Вам также может понравиться